Sei sulla pagina 1di 299

A.

CARBOHYDRATES
1. Describe and classify carbohydrates
Description
Contain C, H and O molecules
Contain a C=O (ketone) and an OH(aldehyde) functional group

Classification
Based on certain properties
The size of the base carbon chain
Location of the CO functional group
Number of sugar units
Stereochemistry of compound

Chemical Properties
Some ( not all ) carbs are reducing substances (donate electrons)
Chemical reduction of other substances
These sugars must contain an aldehyde or ketone group
Reducing sugars
o Glucose
o Maltose
o Lactose
o Fructose
o Galactose
Sucrose is not a reducing substance
2. Describe carbohydrate metabolism
Glucose is primary energy source
Nervous tissue cannot concentrate or store carbohydrates, so a steady supply of
glucose is needed
Once the level of glucose falls below a certain range, normal function is impaired
Carbohydrate Breakdown Ultimate Goal
o Convert glucose to CO2 and water with ATP as a by-product
3. Discuss glycol ysis as it pertains to carbohydrate metabolism and carbohydrate
detection methods

Hydrolysis of glucose by an enzyme into pyruvate or lactate;
This process is anaerobic
4. Fasting blood glucose levels
Hyperglycemic
o Fasting blood glucose >100 mg/dL
Hypoglycemic
o Fasting blood glucose <50 mg/dL

Page 1

5. Describe glycol ysis
Glycolysis the conversion of glucose and other hexoses into lactate or pyruvate
Breakdown of glucose for energy production
6. Describe carbohydrate breakdown

Ultimate Goal
o Convert glucose to CO
2
and water with ATP as a by-product
o Possible channels
o Converted to liver glycogen and stored
o Metabolized to CO
2
and H
2
O
o Converted to keto-acids, amino acids, and proteins
o Converted to fats and stored in adipose tissue
Biochemical pathways in carbohydrate breakdown
o Embden-Meyerhoff pathway
o Converts glucose to pyruvate/lactate
o Primary energy source for humans
o Hexose monophosphate shunt
o Oxidizes glucose to ribose and CO2
o Produces NADPH as an energy source
o Glycogenesis
o Converts glucose to glycogen
7. Describe the role of the liver in maintenance of glucose levels.

The liver maintains the glucose levels by:
o Glycogenesis
Converts glucose to glycogen
Page 2

o Glycogenolysis
Breakdown of glycogen to form glucose
Glycogenolysis occurs when plasma glucose is decreased
Occurs quickly if additional glucose is needed
Controlled by hormones & enzymes
o Gluconeogenesis
Formation of glucose from non-carbohydrate sources, such as
amino acids, glycerol & fatty acids into glucose
Occurs mainly in the liver
During long fasts, gluconeogenesis is required to maintain blood
glucose levels because glycogen stores are up in about 24 hours


During a fast, the blood glucose level is kept constant by mobilizing the glycogen
stores in the liver

Page 3

8. What hormones does the liver use to maintain glucose levels?

Insulin
o Produced by the beta cells of the islets of Langerhans in the pancreas
o Promotes the entry of glucose into liver, muscle, and adipose tissue to be
stored as glycogen and fat;
o Inhibits the release of glucose from the liver
o Insulin secretion controlled by:
Blood glucose level
Certain Amino Acids ie. leucine, & arginine


Glucagon
o Secreted by the alpha cells of the pancreatic islets of Langerhans
o Increases blood glucose by stimulating glycogenolysis and gluconeogenesis
o 2nd most important glucose regulatory hormone
o Referred to as a hyperglycemic agent
o Synthesized in alpha cells of the islets of Langerhans
o Action/Effect of
o Stimuli decreased plasma glucose
o Action
Increases glycogenolysis & gluconeogenesis
Promotes breakdown of fatty acids
Promotes breakdown of proteins to form amino acids
Increases plasma glucose concentration
Somatostatin
o Origin-Delta cells of the islets of Langerhans in the pancreas
o Effect - increase plasma glucose
o Actions
antagonistic to insulin,
Page 4

inhibits endocrine hormones including glucagon & growth hormone
o Inhibits secretion of insulin, glucagon, and growth hormone resulting in an
increase in plasma glucose levels
Other regulatory hormones
o Epinephrine
One of two glucose regulating hormones from the adrenal gland
Origin adrenal medulla
Action/effect
Inhibits insulin secretion & release
Promotes lipolysis
Stimulates glycogenolysis
Immediate release of glucose
Stimuli
Neurogenic - based on physical / emotional stress.
Adrenal tumors
o Glucocorticoids - such as cortisol
Origin adrenal cortex
Effect antagonistic to insulin
increases blood glucose
promotes gluconeogenesis from breakdown of proteins
inhibits the entry of glucose into muscle cells
Stimuli anterior pituitarys ACTH
o Growth Hormone (GH) and Adrenocorticotropic Hormone (ACTH)
Origin anterior pituitary gland
Effect antagonistic to insulin
Increases plasma glucose levels
inhibits insulin secretion
inhibits entry of glucose into muscle cells
inhibits glycolysis
inhibits formation of triglycerides from glucose
Stimuli
decreased glucose stimulates its release
increased glucose inhibits its release
o Thyroid hormones (such as thyroxine)
Origin thyroid gland
Effect
Increases absorption of glucose from intestines
Promotes conversion of liver glycogen to glucose
o Stimuli pituitary glands TSH

9. What hormones does the pancreas produce that regulate carbohydrate
metabolism?

Insulin
o Produced by the beta cells of the islets of Langerhans in the pancreas
o Promotes the entry of glucose into liver, muscle, and adipose tissue to be
stored as glycogen and fat;
Page 5

o Inhibits the release of glucose from the liver
Glycagon
o Secreted by the alpha cells of the pancreatic islets of Langerhans
o Increases blood glucose by stimulating glycogenolysis and gluconeogenesis
Somatostatin
o Synthesized by the delta cells of the pancreatic islets of Langerhans
o Inhibits secretion of insulin, glucagon, and growth hormone resulting in an
increase in plasma glucose levels
10. What impact does cortisol, catecholamine hormones and thyroid hormones have
on glucose levels.

Cortisol
o Secreted by the adrenal glands;
o Stimulates glycogenolysis, lipolysis, and gluconeogenesis
o Increases plasma glucose by decreasing intestinal entry into cells and
increasing gluconeogenesis.
Epinephrine
o Increases plasma glucose by inhibiting insulin secretion.
o Secreted by the medulla of the adrenal glands.
o It stimulates glycogenolysis and lipolysis;
o It inhibits secretion of insulin.
o Physical or emotional stress causes increased secretion of
epinephrine and an immediate increase in blood glucose levels.
Thyroid hormone
o Secreted by the thyroid gland;
o Stimulates glycogenolysis and gluconeogenesis;
o Increases glucose absorption from the intestines
11. Describe the metabolic defect in Diabetes Mellitus?

Glucose does not get into the cells
12. What are typical glucose levels, insulin levels, and ketone levels in this disease?

1) Type I Diabetes Mellitus-
o Glucose levels are increased
o Insulin levels are decreased
o Ketones present

2) Type II Diabetes Mellitus
o Glucose levels are increased
o Insulin levels are normal to decreased
o Glucagon response is decreased
o No ketones present.
13. What is glycosylated hemoglobin?

Glycated hemoglobin is formed from the nonenzymatic, irreversible attachment of
glucose to hemoglobin A1.
Page 6

Measurement of glycated hemoglobin reflects blood glucose levels for the past 2 3
months.
It is useful in monitoring effectiveness of treatment and compliance of diabetic
individual to treatment protocol.
The primary determinant in the rate of hemoglobin A-1c synthesis is the life span of
the Red Blood Cell and the level of average glucose concentration
14. What are the normal glucose levels in a fasting individual?

Normal glucose level (fasting)- 70 to 110 mg/dl
15. What are panic or critical glucose values?

Panic Values- >126 (fasting), >200 (random or glucose tolerance test)
16. Describe the relationship between glucose levels in urine and serum.

Glucose will never be found in urine unless the serum glucose is high enough to
exceed the renal threshold and spill over into the urine
Glucose is filtered by the glomeruli, reabsorbed by the tubules, and normally not
present in urine.
If the blood glucose level is elevated, glucose appears in the urine, a condition
known as glucosuria.
An individuals renal threshold for glucose varies between 160 and 180 mg/ dL.
When blood glucose reaches this level or exceeds it, the renal tubular transport
mechanism becomes saturated, which causes glucose to be excreted into the
urine.

17. Name inherited disorder of carbohydrate metabolism
Glucose 6 phosphatase deficiency AKA von Glerke
Galactosemia
o Characterized by a deficiency or absence of galactokinase,
o Enzyme defect prevents metabolism of galactose.
o Galactose is found in milk as a component of lactose, with galactosemia
generally identified in infants.
18. Why should serum for glucose be removed from the red cells as soon after
collection as possible?

Serum should be removed from RBCs ASAP because the cells will use the
glucose and falsely decrease the glucose level.

19. What anticoagulant preservatives are used for glucose specimens? Why?

Sodium fluoride is the anticoagulant of choice because it inhibits glycolytic
enzymes.

Page 7

20. What is a two-hour post prandial glucose? Why is it performed?

This is a sample taken two hours after eating and can determine how well the body
is using the glucose.
This sample will show insulin function which is the main reason it is performed.

21. Why are D-xylose tolerance tests performed?

This test is done to differentiate malabsorption from pancreatic insufficiency.
22. What is the normal range of CSF glucose?

Normal CSF glucose- 40-70 mg/dl
CSF glucose is 2/3 of the plasma glucose which is due to the glucose entering the
CSF by facilitative transport.
The carrier mechanism is responsible for transporting glucose across the downhill
gradient.
Meningitis can cause a change in the CSF glucose levels.

23. Describe states that result in alteration of serum glucose.

Diabetes mellitus Increases glucose (insulin is not working)
Pancreatitis- Increases glucose (insulin not produced)
Recent meal intake Increases glucose
Fasting Decreases glucose

24. Lactate
The normal end product of glucose metabolism is pyruvate;
Lactate is produced under conditions of oxygen deficit (anaerobic metabolism).
The production and accumulation of lactate in the blood and its measurement aid in
assessing the degree of oxygen deprivation that is occurring.
Change in the blood lactate level precedes a change in blood pH.
Lactate is metabolized by the liver via gluconeogenesis.

25. What are normal glycosylated hemoglobin ranges?

Normal glycosylated hemoglobin range
o 4.5-8.0
High values indicate that the patient has not been following the proper diet




Page 8



Page 9

B. LIPIDS
1. Describe the cholesterol
Function
o Used to manufacture and repair cell membranes,
o Used in synthesis of bile acids and vitamin D
Synthesized in the liver and obtained from the diet.
Precursor for synthesis of bile acids, steroid hormones, and vitamin D
Transport mechanism
o Transported through the blood by LDL (low density lipoproteins)
Storage sites
o Stored in the skin, adipose tissue, and muscle cells.
Esterified cholesterol-2/3 of the total cholesterol is esterified

2. Describe triglycerides
Formed from one glycerol molecule with three fatty acid molecules attached via
ester bonds
Comprises 95% of all fats stored in adipose tissue
Transport mechanism
Transported through the body by chylomicrons and VLDL
3. HDL-cholesterol
Synthesized by the liver and by the intestine
In normal lipid metabolism, HDL removes excess cholesterol from peripheral
tissues and transports it to other catabolic sites providing an antiatherogenic effect..

4. What is the role of lipase?
Lipase and bile acids are used to break down fats in lipid absorption.
Found in pancreas, with lesser amounts in gastric mucosa, intestinal mucosa,
adipose tissue
Clinical significance:
o Increased serum levels in acute pancreatitis occur in 4 8 hours after the
onset of pain, with peak values in 24 hours, and return to normal in 8 14
days.
5. List the bile acids
Cholic acid,
Glycocholic acid
Taurocholic acid
6. Where are bile acids synthesized?
Liver
7. What does the presence of bile acids in serum indicate?
. The presence of bile acids in serum can indicate liver disease.
Page 10

8. What are chylomicrons?
Triglycerides are transported through the body by chylomicrons
Chylomicrons- the largest lipoprotein particles with diameters ranging from 80-1200
nm. They are 90-95% triglycerides, 2-6% phospholipids, 2-4% cholesteryl ester,
1% free cholesterol, and 1-2% apolipoprotein.

9. How does serum appear with increased chylomicrons? increased cholesterol?
increased triglycerides?
Increased Chylomicrons- a creamy layer appears on top of the serum.
Increased Cholesterol- serum appears milky white
Increased Triglycerides- serum appears turbid.
10. Given a value of 38 mg/dl for the HDL, 140 for triglycerides and 210 for total
cholesterol, calculate the LDL and VLDL.

VLDL=triglycerides/5
LDL=total cholesterol- VLDL- HDL
VLDL=140/5 =28
LDL=210-28-38=144
C. LIVER FUNCTION AND HEME-DERIVATIVES
1. Describe liver function in reference to each of the following:

3) Carbohydrate synthesis and metabolism
The liver uses glycogenesis to make glycogen from glucose, simple sugars
and amino acids.
The glycogen produced is stored and used as needed.
The liver also uses glucose for maintenance of mitochondrial NADH and to
generate ATP in the Embden Myerhoff pathway.
4) Protein synthesis and metabolism
The liver makes various proteins (transferrin, prothrombin, and ceruloplasmin)
which are used to transport materials like iron, and copper.
The liver also uses the proteins for nutrition, to regulate oncotic blood pressure
(albumin), and for coagulation.
o These proteins include albumin, HDL, LDL, VLDL, haptoglobin,
angiotensin, erythropoiten, and many others.
o These proteins also are used for nitrogen excretion by processing
ammonia, urea, creatinine and uric acid.
5) Lipid synthesis and metabolism-
This is where cholesterol, triglycerides and phospholipids are synthesized.
Free fatty acids are metabolized in the citric acid cycle into NADH, and bile
acids are produced form cholesterol.
6) Porphyrin synthesis
The liver produces the enzyme aminolevulinic acid synthase which controls
the synthesis of porphyrins which eventually form heme molecules.
Page 11

7) Bile acid synthesis
the bile acids are conjugated with amino acids to form bile salt.
It is synthesized by cholesterol in the bile ducts and ends up in the intestine
where the lipids are digested.
8) iron and vitamin storage
Iron is stored in the liver and transported wherever needed by transferrin, and
vitamins are stored in the liver and available to be used whenever needed.
9) Excretion of metabolic end product and detoxification
It converts ammonia to urea.
10) Bile pigment formation
Bilirubin is the principal pigment in bile and is derived from the breakdown of
hemoglobin when aged red blood cells are phagocytized by the
reticuloendothelial system, primarily in the spleen, liver and bone marrow.

2. Where is bilirubin produced
Bilirubin is produced in the reticuloendothelial system from the breakdown of
hemoglobin from senescent red blood cells
Bilirubin forms a complex with albumin for transport to the liver.
In this form, bilirubin is unconjugated and not water soluble.
3. Describe heme catabolism by the reticuloendothelial system (extravascular
hemolysis). List states associated with increased extravascular hemolysis.
Hemoglobin is broken down extravascularly into globin and heme (iron and
protoporphrin IX)
Protoporphrin breaks down further into unconjugated bilirubin which is carried to the
liver by albumin and conjugated with glucuronyl transferase.
o This is associated with RBC membrane defects and defects in the heme
structure.
4. Identify the function of haptoglobin, hemopexin albumin and methemalbumin in
heme catabolism in intravascular hemolysis.
Haptoglobin binds free hemoglobin and takes it back to the liver so it is not lost in
the urine,
Hemopexin albumin removes circulating heme from the blood and delivers it to the
liver,
Methemalbumin is free heme oxidized and bound to albumin which is carried to the
liver and acts as storage that can be used until enough hemopexin is produced
and made available to the liver.
5. List states associated with increased intravascular hemolysis.
Intravascular hemolysis is associated with increased immunologic processes,
mechanical injury, and toxins.
6. Differentiate between unconjugated and conjugated bilirubin
Conjugated bilirubin
Page 12

o Bilirubin is conjugated in the hepatocyte endoplasmic reticulum with
glucuronic acid to form bilirubin diglucuronide ( conjugated bilirubin).
o The reaction is catalyzed by UDP
o Conjugated bilirubin is water soluble.
o Conjugated bilirubin is excreted into the bile for storage in the gallbladder,
secreted into the duodenum in response to gallbladder stimulation, and
reduced by anaerobic bacteria in the intestine to urobilinogen.
o Some intestinal urobilinogen is reabsorbed;
A portion returns to the liver and some enters the circulation for
excretion in the urine, whereas the remaining portion in the
intestines is oxidized by anaerobic bacteria for excretion in the stool
as urobilin.
o Urobilin is an orange- brown pigment that gives stool its characteristic color.
Unconjugated bilirubin
o Bilirubin is produced in the reticuloendothelial system from the breakdown
of hemoglobin from senescent red blood cells
o Bilirubin forms a complex with albumin for transport to the liver.
o In this form, bilirubin is unconjugated and not water soluble.
7. Describe the process of bilirubin conjugation. What is the role of glucuronyl
transferase and where is this enzyme synthesized? What relationship exists
between enzyme synthesis and neonatal physiologic jaundice?
Hemoglobin is broken down into portoporphrin which is converted into
unconjugated bilirubin which is bound to albumin and carried to the liver where it is
converted to conjugated bilirubin by the enzyme glucuronyl transferase
(synthesized in the liver).

8. What is Kernicterus? Why does it develop?
This is a serious newborn condition that occurs in the central nervous system
because of high bilirubin levels.
It is caused by an under developed blood brain barrier, and because newborns do
not produce enough glucuronyl transferase.
9. Describe excretion of bilirubin and resulting formation of urobilinogen? Why is
urobilinogen normally present in urine and serum?
Conjugated bilirubin is taken to the intestines where bacteria convert it into
urobilinogen.
Urobilinogen is normally present in the urine because a small amount of it is filtered
back to the liver where it is recirculated and sent to the kidney where it is excreted
in the urine.
10. Prehepatic jaundice
Prehepatic jaundice occurs when there is excessive erythrocyte destruction, as
seen in hemolytic anemias, spherocytosis, toxic conditions, hemolytic disease of
the newborn caused by Rh or ABO incompatibility, etc
Page 13

In these cases, the rate of hemolysis exceeds the livers ability to take up the
bilirubin for conjugation.
Prehepatic jaundice is characterized by an increased level of unconjugated bilirubin
in the serum.
11. Hepatic jaundice
occurs when the liver cells malfunction and cannot take up, conjugate, or secrete
bilirubin.
o Gilbert syndrome:
Defect in the ability of hepatocytes to take up bilirubin; due to
transport problem of bilirubin from the sinusoidal membrane to the
microsomal region; characterized by mild increase in serum level of
unconjugated bilirubin
o Neonatal physiological jaundice:
Level of UDP- glycuronyltransferase is low at birth;
Takes several days for the liver to synthesize an adequate amount
of the enzyme to catalyze bilirubin conjugation; causes increased
serum level of unconjugated bilirubin
12. Posthepatic jaundice
Occurs when an obstruction blocks the flow of bile into the intestines.
This is referred to as extrahepatic cholestasis and may be caused by gallstones
obstructing the common bile duct, neoplasms such as carcinoma of the ampulla of
Vater or carcinoma of the pancreas, and inflammatory conditions such as acute
cholangitis or acute pancreatitis.
Posthepatic jaundice is characterized by:
o Significantly increased level of conjugated bilirubin in serum,
o Increased level of unconjugated bilirubin in serum,
o Increased conjugated bilirubin in the urine,
o Decreased urine and fecal urobilinogen
o Stool that appears pale in color.

13. Why are bilirubin determinations performed on amniotic fluid?

D. B. Porphyrins and Heme Derivates
1. What are the porphyrias?
These are deficiencies in the enzyme production that can be acquired or inherited.
They result in increased production of one of the heme precursors to which they
are intermediates.
2. What is the physiologic function of porphyrins.
The most important function of porphyrias are to chelate iron from heme.

Page 14

3. When are plasma hemoglobin levels increased and how are they measured?
The plasma hemoglobin levels are increased during thalessemias and
hemoglobinopathies.
They can be measured by cellulose acetate electrophoresis or citrate agar
electrophoresis.
4. What reagent is used for the detection of urobilinogen? When would
urobilinogen levels be decreased? Increased?
Ehrlrich's reagent is most often used for urobilinogen detection

Increased Urobilinogen levels
o Excess hemolysis,
o Liver damage by hypoxia
o Exposure to various toxic agents.
Decreased Urobilinogen levels
o Obstructive jaundice because there is a limited delivery of bilirubin to the
gut.
5. When would myoglobin be increased? How are serum myoglobin levels
measured?
Myoglobin is increased when there is trauma to skeletal or cardiac muscle.
(myocardial infarction).
Serum myoglobin levels are measured by Electrophoresis.
E. PROTEINS AND NON-PROTEIN NITROGEN
1. What are the functions of proteins in normal physiology?
Proteins function as transport carriers for other substances.
They transport substances to the proper sites for absorption, modification, or other
utilization.
2. Salt fractionation-
The proteins are fractioned out by the use of salts. The salts decrease the water
available for hydration of the hydrophilic groups and cause precipitation of the
globulins.
3. Zwitterion-
An ion that has both positive and negative regions of charge.
4. Zeta potential
This is the potential difference between the negative charges on the surface of the
red blood cell membrane and the cations in the aqueous medium.
Cations are divided into two groups, those that always move with the RBC and
those that can move freely in the medium.
The zeta potential is measured from the boundary of these two cations to the
negative charge on the membrane.
Page 15

5. Pol ypeptide
Amino acids that combine to form proteins which link together to form peptides.
Many peptides linked together form a polypeptide.
6. Oligoclonal banding
Electrophoretic pattern of CSF form patients with multiple sclerosis with distinct
bands in the globulin zone.

7. Briefly describe the Kjeldahl techniques for determination of protein and non-
protein nitrogen.
In this method the serum proteins are precipitated with an organic acid.
The nonprotein nitrogen is removed with the supernatent.
The protein pellet is digested in H2SO4 with heat and a catalyst (cupric sulfate).
Potassium sulfate can also be used to improve the digestion.
The H2SO4 oxidizes the C, H, and S in protein into CO2, CO, H2O, and SO2.
The nitrogen in the protein is then converted to ammonium bisulfite which is
measured by adding alkali and distilling the ammonia into a standard boric acid
solution.
The ammonium borate formed is then titrated with a standard solution of HCL to
determine the amount of nitrogen in the original protein solution.
8. What are the major causes of increased and decreased albumin? Increased and
decreased globulins?


9. What is the theory of refractometry?
Refractometry- the velocity of light is changed as it passes the boundary between 2
transparent layers (air and water) causing light to be bent.
When solute is added to water the refractive index at 20*C of 1.33 is increased by an
amount proportional to the concentration of the solute in the solution.
10. What are major interfering substances in the determination of serum protein by
refractometry?
Interfering substances- Nonprotien solids (electrolytes, urea, and glucose)

11. Name three ways to separate albumins from globulins.
Electrophoresis,
Chromatography
Precipitation

Page 16

12. Discuss the reasons for determining spinal fluid protein and glucose.
13. What are normal values for spinal fluid protein and glucose?
14. What results are expected from spinal fluid in meningitis ?
In meningitis, encephalitis, and neurosyphilis there would be a decreased glucose
level with increased protein levels (IgG).
15. What results are expected from spinal fluid in encephalitis?
In meningitis, encephalitis, and neurosyphilis there would be a decreased glucose
level with increased protein levels (IgG).
16. What results are expected from spinal fluid in neurosyphilis?
In meningitis, encephalitis, and neurosyphilis there would be a decreased glucose
level with increased protein levels (IgG).
17. Discuss the BCG method for determining albumin? Why is the pH important?
BCG (Bromocresol Green) method for determining albumin is a dye binding
procedure where positively charged albumin is attracted to and binds to the anionic
dye.
Once bound to the albumin, the dye has a different absorption maximum than free
dye.
The amount of albumin can be quantitated by measuring the absorbance of the
albumin-dye complex to which it is directly proportional.
The pH must be adjusted on the solution to make the albumin positively charged so it
will bind to the dye.
18. What is Biuret reagent? Explain its function in determination of total protein.
What are the major interfering substances?
The biuret reagent contains sodium potassium tartrate to complex cupric ions to
prevent their precipitation in the alkaline solution, and potassium iodide which acts as
an antioxidant.
In this procedure small peptides react and the color of the chelate produced has a
different shade that seen with larger peptides (color varies from pink to reddish violet
and is measured at 510nm).
Major interfering substances are any compound with 2 or more of the following
groups NHCH2 , and NHCS.
19. What is a protein-free filtrate? List 3 precipitating reagents used.
A protein free filtrate removes proteins from whole blood, serum, urine, or other body
fluids by precipitation with a precipitant and then filtration or centrifugation.
Some precipitating reagents are:
o Tungstic acid
o O-toluidine,
o Horseradish peroxidase,
o Molybdate
Page 17

o Trichloroacetic acid

20. What is Nessler's reagent? In what other reactions may it be used?
Nessler's reagent:
o Double iodide of mercury and potassium.
o This reagent is used in the determination of non-protein nitrogens (NPN).
o It is also used in Nessleration reactions of urea nitrogen.
21. What is an A/G ratio? How is it used diagnostically?
A/G ratio determines how much albumin is in the body compared to globulins.
You use it in correlation with Total protein to determine if albumin or globulins are
low.
F. Specific serum proteins
1. What is alpha-1 antitrypsin? Ceruloplasmin? Name and describe disease
processes involved with each protein.
Alpha-1 antitrypsin
o Acute phase reactant
o Main function
Neutralizes trypsin-like enzymes that can cause hydrolytic damage to
structural protein.
o Disease process:
Severe deficiency of this is associated with severe degenerative
emphysematous pulmonary disease.
Ceruloplasmin
o Copper containing glycoprotein that stores 90% of the total serum copper.
o Disease process:
Low serum concentrations of copper are associated with Wilsons
disease.
2. What is the function of haptoglobin?
Haptoglobin functions to bind free Hemoglobin by the alpha chain.
3. Name three methods for measuring haptoglobin.
3 methods to measure Haptoglobin:
o Starch gel electrophoresis,
o Radial immunodiffusion,
o Immunonephelometric methods.
4. When is haptoglobin decreased? Increased?
Increased Haptoglobin levels:
o Inflammatory conditions,
Page 18

o Burns, and
o Nephrotic syndrome
Decreased Haptoglobin levels:
o Intravascular hemolysis,
o Transfusion reactions,
o HDN,
o Mechanical breakdown of RBC's,
o Athletic trauma.
5. Define Troponin
Troponin- is a complex of 3 proteins that bind to the thin filaments of striated muscle
(cardiac and skeletal) but are not present in smooth muscle.
6. List three isoforms that make up the troponin complex.
o 3 isoforms:
TnT,
TnI,
TnC
7. Describe the advantages of troponin over CKMB and name the method currently
available to measure cTnI.
o Cardiac troponin I is highly specific for myocardial tissue, and because it does
not normally circulate in blood it is 13x more abundant in the myocardium than
CKMB on a weight basis.
o cTnI is very sensitive and can indicate even a minor amount of cardiac necrosis.
o The relative increase of cTnI is greater than that of CKMB.
o cTnI can be measured by:
o Immunoenzymetric assays using 2 monoclonal Ab's directed against different
epitopes on the protein.
G. Protein Electrophoresis
1. What is a monoclonal gammopathy? polyclonal gammopathy?
A monoclonal gammopathy is a sharp narrow band in the late beta or gamma region
that suggests a monoclonal M spike.
The M spike is a spike of one class of Igs that is possibly metastasizing or producing
clones of itself and suggests cancer, possibly multiple myeloma.
A polyclonal gammopathy is a broad gamma band that is increased. It looks
abnormal and is usually caused by an infection. It is made up of more than one
serum protein being increased.
2. Where are the sites of synthesis for the following proteins:
a. albumin- liver
b. alpha-1 globulin- liver
c. alpha-2 globulin- liver
Page 19

d. beta globulin- liver
e. gamma globulins- (IgA, IgG, and IgM)- made by B cells that become plasma
cells.
3. Discuss the clinical picture of multiple myeloma. What are the expected results
of electrophoretic patterns on serum and urine of a myeloma patient?
In multiple myeloma you would see:
o Bence jones proteins found in the urine (free kappa and lambda light
chains)
o "M" spike in electrophoretic pattern
4. Briefly describe serum protein electrophoresis.
Serum electrophoresis
o Serum samples are applied to the cathode end of a support medium strip
that is saturated with an alkaline buffer (8.6). The strip is connected to 2
electrodes and a current is passed. All serum proteins are negatively
charged at the 8.6 pH so they migrate toward the anode end.
5. Name the five bands that occur in serum and list the major proteins that migrate
in the five bands.
6. Diagram a normal pattern as they migrate from anode to cathode in a barbital
buffer and label each peak.

7. What type of electrophoretic patterns can be expected in the following disease
states? Explain the patterns and sketch them.
a. Multiple myeloma

b. Nephrotic syndrome

c. Liver disease

d. Chronic infection

e. Acute phase reaction

f. Malnutrition
Page 20

8. Describe CSF protein electrophoresis.
CSF protein electrophoresis is done with the same technique as serum
electrophoresis, except agarose gel is used most often because it is a high
resolution technique.
9. What bands are normally seen in CSF protein electrophoresis
Oligoclonal bands are: distinct bands seen in the globulin zone and they are
associated with multiple sclerosis (90%), and Inflammatory infectious neurological
disease.
10. What are oligoclonal bands and what disease process are they associated with?
pre-albumin,
albumin,
alpha-1 globulin (antitrypsin),
insignificant alpha-2 globulin,
Beta-1 zone (transferrin)
Beta-2 zone.

11. Describe protein electrophoresis patterns would appear in the following
situations and explain why the pattern appears as it does.

a. Electrophoresis of plasma instead of serum


b. Serum containing alpha-feto protein


c. Fresh serum containing complement


d. Bisalbuminemia


e. Serum containing C-reactive protein

Page 21

H. Creatinine, Bun, Uric Acid

1. What is NPN? Which compounds comprise 50% of the total NPN's?
Non-protein nitrogen (NPN)- nitrogen containing compounds that remain in the blood
sample after the removal of protein constituents.
Urea is the compound that comprises 50% of the total NPNs.

2. What is BUN? What is Azotemia? How is urea nitrogen converted to urea
mathematically?
Blood urea nitrogen (BUN)- nitrogen in the blood in the form of urea. This is a
measurement that is used to analyze the urea level.
Azotemia- an elevated level of urea in the blood.
Bun x 2.14 =urea
3. What is creatine? How is it measured?
Creatine- a compound found in muscle synthesized from several amino acids. It
combines with high energy phosphate to form creatine phosphate which functions as
an energy compound.
We can measure creatine by the J affe method which measures creatine based on
analyzing the sample for creatinine before and after heating in acid solution. The
heating converts the creatine to creatinine and the difference between the two
samples is the creatine concentration.

4. Relate elevations in uric acid to the following disease states:

Primary gout- this is caused by increases of uric acid which cause sodium urates
to precipitate in the joints. This can be caused by overproduction of uric acid,
drugs, and alcoholism.
Secondary gout- this gout is formed as a secondary infection caused by a larger
problem like leukemia.
Leukemia- this causes the increased breakdown of cell nuclei caused by
chemotherapy which causes the uric acid levels to increase.
Polycythemia- this causes the increased breakdown of cell nuclei caused by
chemotherapy, much like leukemia does, which also increases the uric acid
levels to increase.
Glomerular nephritis- in this disease the nephrons of the glomerulus are
damaged in the kidney which causes poor filtration and increased levels of the
uric acid.
Page 22

Multiple myeloma- this disease is also treated with chemotherapy which breaks
down the nuceli and causes the uric acid to increase much like leukemia and
polycythemia.

5. Relate uric acid production to purine catabolism.
Uric acid is formed from the catabolism of purines like adenosine and guanine in the
liver. This uric acid is transported by the plasma from the liver to the kidney where it
is filtered by the glomerulus into the proximal tubules where most is reabsorbed and
only small amounts are secreted into the urine.
6. What is the clinical significance of BUN? (renal, prerenal, postrenal). What
creatinine values are expected in these conditions?
BUN- (direct urea measurement from serum or plasma)
It is used extensively in the determination of renal function.
BUN CREATINE BUN/CR
RATIO
Pre-renal- Increased Normal Increased
Renal- Increased Increased Normal
Post renal- Normal Increased Decreased

7. What are normal values for BUN and creatinine? What is the normal ratio of BUN
to creatinine? When is the ratio altered?
Normal BUN- 7-18mg/dl
Normal Creatinine- 0.5-1.2mg/dl
Normal BUN/Creatinine ratio- 10:1-20:1
The ratio is altered in:
o Low protein uptake
o Acute tubular necrosis
o Severe liver disease
Diagram and describe the Berthelot reaction for BUN. Berthelot Reaction for
BUN-
o Urea is hydrolyzed with urease, and the ammonia ion formed is reacted with
phenol and hypochlorite in alkaline medium to form indophenol.
Nitroprusside is used to catalyze the reaction.
o Absorbance of dissociated indophenol (blue chromogen) is measured at
560nm.
o REACTION:
NH4 +5NaOCC+2 phenol indophenol +5NaCl +5H2O
8. What is the purpose of the following reagents?
o urease- used ot prepare the stock suspension
Page 23

o sodium nitroprusside- catalyzes the reaction
o phenol- converted to indophenol
o alkaline hypochloride- aids in conversion of phenol to indophenol.
9. What are some advantages and disadvantages of this method? What
anticoagulant must be avoided when using any urease method?
o Advantages- can use serum, plasma, or urine.
o Disadvantages- contamination of urine with bacteria is common and can
cause decreased urea and formation of ammonia.
10. What kidney functions do the following clearance tests measure?
inulin- Reference substance for measuring GFR (glomerular filtration rate)
creatinine- universely used in assessment of GFR.
urea- not useful in monitoring GFR, but serum urea may provide useful clinical
metabolic
information.
p-amino-hippurate- reference substance for the measurement of renal plasma flow.
11. Diagram and describe the Jaffe reaction for creatinine. What substances give
false positive reaction?
J affe reaction-
o The reaction occurs between creatinine and the picrate ion formed in the
alkaline medium and a red-orange adduct develops. Teh observed rate of
the hydroxyl ion concentrations over a broad range of picric acid
concentrations. This is measured spectrophotometricaly at wavelengths of
485-520nm.
Substances that give false positives are:
Protein,
Glucose,
Vitamin C (ascorbic acid),
Acetone,
cephalosporin
12. Why is creatinine preferred to urea for clearance tests? What data are necessary
to calculate creatinine clearance? Write the formula. What are the normal values
for creatinine clearance?

Creatinine is more specific for kidney function than urea is.
Data necessary for calculating creatinine clearance:
o Urine volume,
o Creatinine concentration. in urine,
o Creatinine concentration in plasma
Creatine Clearance =(Urine Cr x Urine volume)/ (Plasma Cr)
Normals: Male=97-137 ml/min Females =88-128 ml/min

Page 24

13. A creatinine clearance was performed on a male patient 1.5m tall and weighing
65kg. His blood contained 2.5 mg/dl creatinine. The urine creatinine was 50
mg/dl and the urine volume was 300 ml/4hrs. What was the creatinine clearance
for this man?
Creatine Clearance =(50 x 300)/ 2.5 x 1.76/ 1.60 (body surface area *see chart)

14. What is creatinine? What is the normal range? What single disease state is
associated with elevated creatinine.
Creatinine is a compound formed when creatine or creatine phosphate
spontaneously loses water or phosphoric acid.
It is excreted into the plasma at a relatively constant rate in a given individual and
excreted in the urine. Its decrease is associated with renal dysfunction as in
glomerulonephritis.
15. Diagram the reaction, list reagents used and describe the principle of the
oxidation reduction method for uric acid using phosphotungstic acid.
This is the most common method used.
It is based on the oxidation of uric acid in a protein-free filtrate with subsequent
reduction of phosphotungstic acid to tungsten blue.
It uses Na carbonate to provide the alkaline pH necessary for the color
development.
The blue color produced can be intensified by adding cyanide or by keeping the
proper pH.

I. Miscellaneous Proteins
1. Describe the method for detecting phenolketonuria? What enzyme deficiency
results in phenolketonuria?
Phenolketonuria results from a total absence of or absence of activity of the enzyme
phenylalanine hydrolase (AKA phenylalanine-4-mono-oxygenase).
You can use the Guthrie test to detect PKU.

2. What are cryoglobulins? How are they measured?
Cryoglobulins are serum protein that precipitates at temperatures lower than body
temperature.
3. What is alpha-fetoprotein? What does its presence signify? How is it detected?
Alpha-fetoprotein is a globulin protein synthesized in the fetal yolk sac and then by
the parenchymal cells of the liver.
Page 25

It is measured to determine if there is increased passage of fetal proteins into the
amniotic fluid.
We also measure the alpha-fetoprotein levels in association with spinabifiida, renal
tube defects, and general fetal distress.
4. What is carcinoembryonic antigen? What types of tumors is it most frequently
associated with? What types of methods are used to measure it?
Carcinoembryonic anitgens (CEA) are glycoproteins which are associated with
numerous cancers (colon, lung, pancreas, stomach, or breast tissue tumors)
5. For each of the following tumor markers describe the types of tumors they are
most often associated with and how they are measured.
CA125- ovarian cancer.
PSA- (prostate specific antigen)- Prostate cancer
J. ENZYMES

1. (L) Give the substrates for the following: LD, CK, AST, ALT, GGT, CK-MB
LD Lactate
o Catalyzes oxidation of Lactate to Pyruvate and the reverse reaction of
Pyruvate to Lactate
o ischemia,
o myocarditis,
o cardiac congestion;
CK Creatinine
o Catalyzes the reversible phosphorylation of ATP
o Muscular dystrophy,
o muscle malignancies,
o heart disease,
o thyroid disease,
o CNS disease.
AST Aspartate
o Transfers amino acids
o This is higher in neonates due to their immature liver
o Liver disease (20-100 times normal in hepatitis),
o carcinoma,
o cirrhosis,
o liver disease
o heart disease,
o muscle disease,
o gallbladder disease,
o AMI,
o pulmonary embolism
Page 26

ALT Alanine
o Catalyzes the transfer of an amino group of alanine to alpha-ketoglutarate
o Enzymatic-UV Monitoring
o Liver disease,
o carcinoma
o gallbladder disease,
o cirrhosis,
o hepatotoxicity
GGT Glutathione
o Transfers gamma-glutamyl residue from gamma-glutamyl peptides to amino
acids, water, and other small peptides
o Liver disease,
o obstructions of the internal liver or gallbladder,
o alcoholism,
o pancreatic problems
CK-MB Creatinine
o Catalyzes the reversible phosphorylation of ATP
o Myocardial problems
2. (L) Define:
a. Isoenzyme-
o one of several forms in which an enzyme can exist in various tissues.
o Although they are similar they can be separated from each other by special
chemical tests (electrophoresis) to give more specific information.
b. Coenzyme-
o These are enzyme activators that are usually heat stable and of low
molecular weight.
o When these are combined with an inactive protein called an apoenzyme they
form an active compound or a complete enzyme called holoenzyme.
c. Catalyst-
o Substance that speeds up the rate of a chemical reaction without itself being
permanently altered of used up in the reaction.
o They are effective in small quantities and are not used up in the reaction.
They can be recovered unchanged.
d. Activator
o Substance in the body that converts an inactive substance into an active
agent.
o Example: the hydrogen ions on pepsinogen converting it to pepsin.
e. Inhibitor-
o Chemical substance that stops the enzyme activity.
f. Hydrolase-
o Enzyme that causes hydrolysis. These catalyze bond cleavage by the
addition of water.
g. Oxidoreductase-
Page 27

h. Enzyme that removes electorns and their corresponding electrons.
i. Transferase-
o These enzymes move chemical grouping from one compound to another.
3. Relate amylase and lipase activity to the following disease states:
a. Acute pancreatitis- both increase in this.
b. Malabsorption- increased in both.
c. Chronic pancreatitis- increased in both
d. Pancreatic carcinoma- increased in both
e. Cystic fibrosis- increased in both because it leads to malabsorption
4. What are the sources of acid phosphate in the body? What are normal ranges for
acid phosphatase in males and females?
o Acid phosphate is found in most tissues in the body like bone, bone marrow,
liver, spleen, RBCs, platelets, and in the highest concentration in the prostate
gland of the male.
5. 8. (L) Explain the clinical significance of alkaline phosphatase in the following
disease states:

a. Obstructive jaundice- increased ALP levels
b. Parenchymal jaundice- increased ALP levels
c. Paget's disease- increased ALP levels
d. Hyperparathyroidism- increased ALP levels
e. Pregnancy- increased ALP levels
6. List enzymes elevated in hemolysis? What enzyme might be depressed with
refrigeration and freezing?
Enzymes elevated in hemolysis:
o CK, LD, AST (aspartate transferase), ACP (acid phosphatase), ALP
(alkaline phosphatase), and LIPASE
Enzymes depressed by refrigeration:
o LD (occurs at 4*C within 24 hours), and LIPASE (occurs if stored at 4*C for
3 weeks)
7. What is cholinesterase? Why is it important in presurgery cases?
Cholinesterase: an enzyme found in RBCs, lungs, spleen, nerve endings, and brain. It
is responsible for the prompt hydrolysis of acetylcholine released at the nerve endings to
mediate transmission of the neural impulse across the synapse. The degradation of
acetylcholine is necessary to the depolarization of the nerve so that it can be repolarized
in the next conduction.
o This is important to measure cholinesterase in presurgery to determine the
amount of succinyl dicholine (muscle relaxer) that can be given in surgery without
complications. (You can only give the amount of succinyl dicholine to the patient
that the patients cholinesterase can rid their body of.)
Page 28

8. What is the relationship of amylase and lipase in pancreatic disease? Why are
both tests necessary in the monitoring of the disease?
o Amylase and Lipase are both elevated in pancreatic disease.
o You must monitor both amylase and lipase in pancreatic disease because:
Amylase is more sensitive, but less specific (b/c also found in other parts
of the body)
Lipase is more specific, but found in small quantities (less sensitive b/c
only found in the pancreas)
9. Discuss the following in relation to amylase activity: activators, pH,
temperature.
o Activatiors: Calcium and Chloride ions
o pH- optimal is 6.9 - 7.0
o Temperature- optimal is 37-40*C
10. Discuss prostatic disease and acid phosphatase levels in serum.
Total activities of ACP may reach 40-50 times the normal in severe stages of
prostate cancer. If the carcinoma is highly localized to the prostate there may only
be slight increases in ACP activity. In benign hypertrophy of prostate, enzyme
levels are normal.
11. How is the L(+) tartrate utilized in the determination of acid phosphatase?
Tartrate inhibits the activity of non-prostatic ACP so that specificity is enhanced
when it is used.
12. Explain heat separation of alkaline phosphate isoenzymes. How does heat effect
the liver fraction? bone? placenta?
Heat separation of alkaline phosphatase Isoenzymes
ALP activity is determined by measuring ALP before and after heating serum at
56*C for 10 minutes.
Placental ALP is the most heat stable followed by intestinal, liver, then bone.
Placental ALP will resist heat denaturation at 65*C for 30 min.
If the residual activity after heating is <20% of the total prior to heating then it is
bone phosphatase.
If the residual activity after heating is >20% of the total prior to heating then it is
Liver phosphatase.
K. Enzyme Electrophoresis

1. List the CK isoenzymes. Describe the makeup of each fraction and organs
associated with each fraction. How are isoenzymes separated?
CK-1 ( brain, brain subunits)- brain, prostate, uterus, bladdar, placenta
CK-2 (muscle, brain subunits)- heart muscle and skeletal muscle
CK-3 (muscle, muscle subunits)- sketal muscle and heart muscle
Page 29

2. How can hemolysis affect the LDH electrophoretic pattern? What is the clinical
significance of this?
Hemolysis can cause LDH electrophoresis to have an LD-1 to LD-2 flip.
Using a hemolyzed sample would cause the results to have a LD-1 to LD-2 flip as
seen in cases of Myocardial Infarctions and Hemolytic anemia.
L. ELECTROLYTES AND TRACE ELEMENTS
1. For each of the following give normal range, panic values, categorized as anion
or cation if applicable and categorize as intracellular or extracellular if
applicable.
Potassium Cation
Sodium Cation
Calcium Cation
Magnesium Cation
Bicarbonate (HCO
-
3
)
Anion
Chloride (Cl) - Anion
2. Describe the relationship between electrolyte balance and water balance.
Include the roles of the kidney, hypothalamus, ADH and the renin-aldosterone
system.
The electrolyte balance and water balance are directly related.
The plasma sodium concentration depends greatly on the intake and excretion of
water.
If the sodium increases it stimulates thirst which will increase the intake of water,
and the kidneys have the ability to conserve or excrete large amounts of sodium
depending on the blood volume which is directly related to the water volume.
The excretion of water is largely affected by the ADH (which is secreted from the
hypothalamus) release in response to the increase in blood volume.
The renin-aldosterone system acts in the kidney to increase the retention of
sodium and increase the excretion of potassium which will eventually increase
the blood pressure by using the electrolyte to balance the water level.
3. List anticoagulants of choice and the effect of hemolysis if any on the following
ions: K, Na, Ca, Mg, HCO3, Cl, P04, Fe.
All electrolytes should be determined using serum or heparinized plasma.
o K- hemolysis increases K levels
o Mg- hemolysis increases Mg levels
o HCO3- hemolysis increases HCO3 levels
o P04- hemolysis increases PO4 levels
o Fe- hemolysis increases Fe levels

o Na-hemolysis does not effect this significantly
Page 30

o Ca- hemolysis does not effect this significantly
o Cl- hemolysis does not effect this significantly

4. Identify the major functions of sodium, chloride, bicarbonate, and potassium.
Sodium- this electrolyte largely determines the osmolality of the plasma.
Chloride- this electrolyte maintains the electrical balance by balancing the
sodium charge, and using the chloride shift with bicarbonate.
Bicarbonate-this electrolyte is used to maintain the acid base balance and buffer
the blood.
Potassium- this electrolyte regulates the neuromuscular excitability, contraction
of the heart, ICF volume, and hydrogen ion concentration (pH).
5. Define and list conditions associated with each of the following:

Hyponatremia- this is decreased levels of sodium in the blood and is associated
with the blood volume status. It results from sodium loss in excess of water loss.

Hypernatremia- elevated levels of sodium in the blood and is associated with
increased sodium concentration because of excess water loss. It can be caused
by increased sodium intake or decreased water intake.

Hypokalemia- decreased levels of potassium in the blood and is associated with
GI or urinary loss of potassium , or with increased cellular use of potassium. This
can be caused by vomiting, diarrhea, etc.

Hyperkalemia- increased levels of potassium in the blood and is associated with
diabetes mellitus, or metabolic acidosis.
6. What disease process results with increased bicarbonate? decreased
bicarbonate?
Metabolic acidosis is related to the decrease in bicarbonate.
Metabolic alkalosis is related to the increase in bicarbonate.
7. Define titration. Explain its application to chloride methods.
Titration is the diluting out of a sample with a liquid reagent of a known strength
and measuring the volume necessary to convert the sample through a given
reaction.
In the chloride titration method the chloride ions combine with the mercuric ions
to form soluble and undissociated mercuric chloride. The proteins in the serum
are precipitated with the tungstic acid and an aliquot of the filtrate is titrated with
an acidic solution of mercuric nitrate using a color indicator. This color indicator
turns violet-blue at the first excess of mercuric ion.
Page 31


8. List disease processes in which hyperchloremia and hypochloremia occur but
sodium is normal.
Hyperchloremia- this is an increase of serum chloride and occurs in situations
where there is an excess loss of bicarbonate ion due to GI losses, RTA, or
metabolic acidosis.
Hypochloremia-this is an decrease of serum chloride and occurs with the excess
loss of chloride from prolonged vomiting, diabetic ketoacidosis, or aldosterone
deficiency.
9. List causes of an increased anion gap? decreased anion gap?
Decreased anion gap is rare, but may be seen in multiple meyloma because of
abnormal proteins. It can also be caused by instrument error.
Increased anion gap- this may be caused by uremia, ketoacidosis (seen in
starvation or diabetes), posioning due to ingestion of substances like methanol or
ethylene glycol, lactic acidosis, or severe dehydration which causes increased
plasma proteins or instrument error.
10. Explain why stock standards of sodium and potassium are kept in plastic
containers.
Stock standards of Na and K are kept in plastic containers because glass
containers leach out the Na and K from the sample.
11. What is an " anion gap" . List formulas for its determination.
Anion gap- the difference between unmeasured anions and unmeasured cations.
It is useful for indicating an increase in one or more of the unmeasured anions in
serum.
AG =Na - (Cl +HCO3)
AG =(Na +K) - (Cl +HCO3)
12. Why is an anion gap routinely performed on all sets of electrolytes? What is an
unacceptable gap? What is standard operating procedure when an anion gap is
unacceptable?
o An anion gap is routinely performed on all sets of electrolytes because it is useful
in indicating an increase in one or more of the unmeasured anions in serum, and
for QC on th analyzer (an abnormal gap can indicate an analyzer problem if
performed on a person in good health).
o An acceptable gap is 10-20:1, so greater than or less than that would be
considered unacceptable and would need to be rerun.

13. Describe the impact of each of the following on serum potassium levels.
administration of insulin- decrease serum K levels (increases the cellular uptake
of K)
Page 32

acidosis- increase serum K levels (excess H enters cell to be buffered and
causes K to leave the cell to maintain electro neutrality)
alkalosis- decreases the serum K (increases the cellular uptake of K)
14. Discuss the following factors influencing serum calcium and phosphorus levels:
parathyroid hormone- this hormone is used to increase the absorption of calcium
and increase the excretion of phosphorus. To increase calcium it breaks down
the bone to release Ca (bone resprption), it conserves Ca by increasing the
tubular reabsorption in the kidney and it stimulates the renal production of vitamin
D which also increases the Ca absorption. To decrease phosphorus the blood
concentration the PTH increases the renal excretion.
calcitonin- this hormone is used to decrease calcium levels and increase the
phosphorus levels which inhibits the actions of PTH and vitamin D.
Vitamin D (calcitriol)-when the calcium is decreased or the phosphorus is
increased this hormone is used to increase calcium by aiding the effects of PTH
by causing more calcium to be stored or released. It decreases phosphorus by
increasing the absorption of it in the intestines and increasing the reabsroption in
the kidneys.
plasma proteins-Albumin is the plasma protein that maintains the appropriate
fluid in the tissues, and it binds various substances in the blood like calcium.
serum pH- a decrease in pH will increase the phosphate levels in the serum like
seen with antiacids.
15. Discuss calcium, phosphorus and PTH levels related to the following disease
states:
bone disease-calcium will be normal to low, phosphorus will be normal to low,
and PTH will be normal to high.
malabsorption- calcium will be decreased, Phosphorus will be decreased, and
PTH will be increased.
renal failure- calcium will be low to normal, phosphorus will by high, and PTH will
also be high.
liver disease- calcium will be decreased, Phosphorus will be decreased, and PTH
will be increased.
primary hyperparathyroidism-Calcium will be Increased, Phosphorus will be
decreased, and PTH will be high.
secondary hyperparathyroidism-Calcium will be decreased, Phosphorus will be
low to normal to high, and PTH will be increased.
primary hypoparathyroidism-Calcium will be decreased, Phosphorus will be
increased, and PTH will be decreased.
secondary hypoparathyroidism-Calcium will be decreased, Phosphorus will be
increased or decreased, and PTH will be decreased.
Page 33

16. What is the physiologic relationship between calcium and phosphorus? Why?
What are the physiologic functions of calcium and phosphorus?
Calcium and phosphorus are inversely related in the serum because phosphate
is an intracellular anion, and calcium is an extracellular cation..
Calcium functions in bone matrix, as an enzyme activator, in coagulation and
complement, and in muscle contraction.
Phosphorus functions in production of ATP, GTP, CTP, UTP, and DNA
structures, as a major body buffer, and in the bone matrix.
17. What is the relationship of calcium levels to alkaline phosphatase activity?
Alkaline phosphatase levels are increased in periods of bone growth or
reconstruction which uses up the calcium and causes the serum levels of calcium
to decrease. They both also function in bone matrix.

18. Describe three forms of body calcium. To which form is PTH most sensitive?
How can only " active" calcium be measured?
Calcium forms- free-ionized Ca, protein bound calcium, and as complexed salts.
PTH is most sensitive to ionized calcium. To measure only the active calcium you
must measure it under anaerobic conditions because an increase in pH can cause
the protein bound Ca to increase which decreases the ionized Ca, and decreasing
the pH can cause the protein bound Ca to decrease and the ionized Ca to increase.
19. Define and describe tetany? What are the relationship of magnesium and
calcium to tetany.
Tetany- irregular muscle spasms.
Calcium- a rapid decrease in ionized calcium concentration will cause tetany.
Magnesium- this is required along with ATPase for normal Ca uptake following a
contration. It is also required for muscle cell stimulation by regulating the
acetylcholine which is a potent neurotransmitter.
20. Describe the relationship between parathyroid hormone and magnesium levels.
The parathyroid hormone increases the renal absorption of magnesium and
enhanses the absorption of magnesium in the intestine. PTH regulates Ca, Na, and
Mg.

21. What is the physiologic role of magnesium? Where is magnesium stored in the
body?
o Magnesium functions as a cofactor for more than 300 enzymes including
those important in glycolosis, transcellular ion transport, neuromuscular
transmission, synthesis of carbohydrates and many others.
o Magnesium is stored in the bone (53%) and the rest (46%) is in the muscle,
soft tissue and other organs.
Page 34


22. Discuss the following as they pertain to magnesium:
o alcoholism- people who are alcoholics tend to have diets deficient in
magnesium or have problems with malabsorption (hypomagnesemia).
o malabsorption- this causes a decrease in magnesium because it is not
absorbed (hypomagnesemia).
o magnesium sulfate therapy- this is given parenterally to severely ill patients.
o secondary hypoparathyroidism- this may cause an increased renal excretion
of magnesium due to an excess of calcium ions.
23. What is the physiologic function of iron? How is iron transported in the body?
How is iron stored in the body?
Iron functions as part of heme in hemoglobin. It is transported by transferrin and
stored in the body as ferratin and hemosidrin.
24. Describe iron levels, %Saturation, TIBC, and ferratin levels in the following
disease states:
iron deficiency anemia-the % saturation is decreased, TIBC is increased, and ferratin
is decreased.
anemia of chronic infection- the % saturation is normal, TIBC is decreased, and
ferratin is normal to increased.
hemochromatosis- the % saturation is increased, TIBC is increased, and ferratin is
increased.
25. What is the physiologic function of copper? What is Wilson's disease?
ceruloplasmin?
Copper functions as an enzyme indicator, it acts on ferroxidase, and it acts it is a
component of enzymes or proteins involved in redox reactions.
Wilsons disease- also known as hepatolenticular degeneration is a genetically
determined defect in ATPase where the copper is transported normally from the
intestine into the liver, but cannot be transported from the liver into the bile.
26. What is the physiologic function of zinc? How is it measured?
Zinc functions as a cofactor for more than 300 enzymes. It can be measured by
atomic absorption.
27. Define:
total iron binding capacity- (amount of transferrin bound already)
An estimate of serum transferrin levels obtained by measuring the total iron binding
capability of a patients serum. Since transferrin represents most of the iron binding
capacity of serum TIBC it is generally a good estimate of serum transferrin levels.
% saturation- (transferrin saturation in the patients sample)
% saturation =total iron/TIBC x 100
Page 35

unsaturated iron binding capacity- (the amount of sites available to bind iron in
patients sample)
UIBC =iron added excess iron
Serum iron =TIBC UIBC
latent iron binding capacity- (estimate of non-reacting iron bound to transferrin)
28. What is the source of blood ammonia? How is it detoxified? What disease
processes are associated with increased ammonia? How is ammonia related to
Reye's Syndrome?
Blood ammonia arises or comes from the deamination of amina acids through the action
of degestive and bacterial enzymes on proteins in the intestinal tract. Ammonia is
released from metabolic reactions that occur in skeletal muscles during excercize.
Ammonia is detoxified by the liver in the urea cycle where ammonia is converted to urea.
Hepatic failure, Reyes syndrome, and urea cycle enzyme deficiencies are associated
with increased ammonia.
Reyes syndrome is usually following a viral infection and it uses ammonia levels to
correlate with the severity of the disease and prognosis.

29. What is the purpose of renal synthesis of ammonia?
The kidney synthesizes ammonia to compensate for metabolic acidosis.
30. What are the three ketone bodies. When are they formed? List disease
processes associated with increased ketones.
o acetone, acetoacetic acid, and beta-hydroxybutyric acid.
o These are formed as a product of incomplete fat metabolism, and are associated
with diabetes mellitus, starvation, and prolonged vomiting.
31. What is lactic acidosis? When does it occur?
o Lactic acidosis- an increase or accumulation of lactic acid in the blood.
o This occurs if there is improper oxidation of skeletal muscle and other tissues.
32. What role does hemoglobin play as an important buffer system in the body?
What is carbonic anhydrase? What is chloride shift?
Hemoglobin buffers the blood by delivering oxygen to the tissues and then taking
the carbon dioxide to the lungs to be exhaled.
Carbonic anhydrase- this enzyme catalyzes the reaction of carbon dioxide to
bicarbonate and Hydrogen ion.
The chloride shift- the carbon dioxide goes into the RBC and forms carbonic acid,
this acid splits into hydrogen ion and bicarbonate. The bicarbonate leaves the cell
and makes it more negative outside the cell, and more positive on the inside
because of the hydrogen ion. At this point the chloride shifts into the cell to balance
the electorneutrality of the cell.
Page 36

33. What are the main factors which influence the oxygen binding ability of
hemoglobin? Specifically how do acidosis and alkalosis affect O2 saturation?
If there is increased oxygen then the hemoglobin binds more oxygen and if there is
less oxygen the hemoglobin picks up less oxygen and wants to hold on to it. The
ability for hemoglobin to bind oxygen depends mostly on the availability of oxygen.
Acidosis- the pH drops drastically and increases the hemoglobin affinity for O2.
Alkalosis- the pH is increased and it decreases the hemoglobin affinity for O2.
34. Why is heparin the anti-coagulant of choice for pH and blood gas work? How
does it work?
Heparin is used because it holds the ph constant in blood and prevents the change in
gals levels in the sample.
35. Why are blood gas specimens placed in ice immediately after collection?
Blood gas specimens are placed on ice immediately after collection because:
The pH decreases with time if it is not placed on ice immediately after drawn. The lower
temperatures prevent the cells from undergoing glycolysis.
IX. ENDOCRINOLOGY

36. Give expected T4, T3 uptake, FTI and TSH levels in the following:

37. What is the function of the thyroid hormones in body metabolism? Describe the
following disease processes:
The thyroid hormone function is to stimulate the metabolism. Without the thyroid
hormone the metabolism will decrease and the patient will experience obesity, mental
retardation, edema (water in the tissues), decreased body temperature, and anemia.
Page 37

cretinism- condition where the patient has a dysfunctional or no thyroid causing mental
retardation and death. Autosomal recessive.
juvenile myxedema-A dysfunction of the thyroid after birth that has very severe
consequences. All 50 states require a T4 screening test for neonates.
adult myxedema- A dysfunction of the thyroid later in life (>30 years) causing hair loss,
dry skin, yellow pallor, thick tongue, and arterosclerosos.
endemic goiter- an enlarged thyroid due to lack of iodine.
hashimoto's disease- the most common form of thyroid disease occurring mostly in
women 40to 60 years old. Treat with thyroxine. Autoimmune disease.
grave's disease- the patient has and AB to TSH receptor on the thyroid gland. This AB
causes the thyroid gland to think it is TSH and start to release T3 and T4 even though
there is a decrease in actual TSH.
thyroid tumors (goiters)- Tumors that cause the dysfunction of the thyroid gland allowing
abnormal amounts of hormone to be released.

38. What are the roles of TSH? What amino acids are necessary for synthesis?
Body temperature stimulates the thyroid hormone which stimulates the
hypothalamus and causes it to release TRH to the pituitary gland. The TRH
causes the pituitary gland to release TSH to stimulate the thyroid gland which then
releases T3 and T4. TheT3 and T4 cause the body temperature to rise and act as
the shunt to tell the hypothalamus to stop making the TRH and the pituitary to stop
making TSH.
39. What is thyroglobulin? thyroid binding globulin?
Thyroglobulin- an iodine contaning protein secreted by the thyroid gland.
Thyroid binding globulin- A protein that binds thyroid hormones. The TBG assay is
used to confirm results of T3 and T4, or abnormalities it the relationship of the TT4
and T3U test. They can also be used as tumor markers for thyroid cancer.
40. Briefly describe each of the following groups synthesized by the adrenal cortex.
What molecule is necessary for their synthesis?
glucocorticoids- (cortisol) these stimulate gluconeogenesis to increase glucose,
encourage the glycogen production and release from the liver, block epinephrine,
act as and anti-inflammatory, inhibit WBC migration, phagocytosis, increase
hematopoiesis and stabilize lysozome.
mineralocorticoids- (aldosterone) these regulate potassium metabolism and
regulate extracellular fluid volume. They also control water secretion by regulating
Na absorption and K secretion.
androgens- (testosterone, dehydroepiandrosterone, dehydroepiandosterone
sulfate) These function in spermatogenesis and in the formation of the secondary
male sex hormones.
estrogens- (estrogen, estradiol, estriol) these function in ovulation, embryo
preparation for implantation, and menstruation regulation
Page 38

41. Describe the feedback mechanism for cortisol. What is the function of cortisol?
What is the role of ACTH?
The hypothalamus is stimulated by low glucose to secrete CRH which causes the
anterior pituitary to secrete ACTH to help make 11-deoxycortisol. Cortisol is
formed and is found bound to transcortin or as free cortisol. The free cortisol
stimulates he glucose production and when the need is met it acts as a shunt to tell
the hypothalamus to stop making CRH.
42. Diagram the renin-angiotensin-aldosterone system. What is the purpose of this
system?
The afferent arterioles secrete rennin when there is a decrease in blood pressure.
The rennin converts angiotensinogen to Angiotensin I which is converted to
Angiotensin II. Angiotensin II stimulates aldosterone to be produced and secreted
to increase the blood pressure by retaining sodium to passively hold water and
increase the blood volume which increases the blood pressure.
43. What hormones make up the 17-hydroxysteriods? the 17-ketosteroids? Why are
these tests performed?
Cortisone, cortisol, and 11-dehydroxycorticoids make up 17-hydroxysteroids.
Androgens make up the 17-keytosetriods.
These tests are performed to observe the levels for glucosteriods, testosterone,
and other androgens.
44. What are the catecholamines? Where are they synthesized? What is the
precursor amino acid? What is their basic function?
Catecholamines are epinephrine, norepinephrine, and dopamine. They are
synthesized in the adrenal medulla of the adrenal glands. The precursor amino acid is
tyrosine. Their basic function is to increase the blood pressure and heart rate when the
body is excited in the fight or flight mechanism. They have the ability to break down
adipose tissue and glycogen to be used for energy in the fight or flight mechanism.
45. Describe catecholamine levels in the following diseases:
Pheochromocytomas-Tumors of the adrenal medulla or the sympathetic ganglia of the
adrenal glands which cause overproduction of the catecholamines.
Infantile Neuroblastoma- Tumors of the adrenal medulla that cause an increase in
catecholamines.
46. What is the function of prolactin? When is it measured?
Prolactin acts to initiate lactation and maintain it in a pregnant female. It is
measured to evaluate pituitary tumors, galactorrhea, and fertility.
47. Describe levels of FSH and LH during follicular development, ovulation and the
luteal phase? Why and how are these hormones measured?
FSH increases during follicular phase, goes down and rises again just after
ovulation, and drops slowly toward luteal phase.
Page 39

LH is baseline in follicular phase, increases in ovulation, and decreases back to
baseline during the luteal phase.
These hormones are measured by RIA to assess pituitary and gonadal axis,
fertility problems, to investigate puberty problems (late puberty), and to
investigate pituitary tumors.
48. What is the function of gastrin? How is it related to Zollinger Ellison Syndrome?
Gastrin functions to secrete HCl to digest food, pepsin, intrinsic factor, pancreatic
hormones, and bile from the liver.
Zollinger Ellison (ulcers in the stomach or duodenum) syndrome has increased
Gastrin levels, so it can be separated from other peptic ulcers because they dont
increase in production of gastrin.
49. What is the function of the androgens? How are they measured? How is
testosterone measured?
Androgens function in spermatogenesis and formation of secondary sex
characteristics of males. Androgens like testosterone are measured by RIA.
50. Briefly describe the function of the estrogens and progesterone. What test is
used for total urinary estrogens? What is its principle?
Estrogen and progesterone both function in getting the uterus ready for embryo
implantation.
The Brown method (a colorimetric method using Kober reagent to look for a pink
color indicating that estriol is present) is used to measure total estrogen levels.
51. What is the primary site of synthesis of estradiol, and esterone? Explain why
these two estrogens are measured?
Estradiol is produced in the ovary of a pre-menopausal woman.
Esterone is produced in the adrenal cortex of the adrenal glands of a post-
menopausal woman.
Both are measured to determine menstrual cycle disorders and ovulation in
women.
52. What does the T3 uptake measure? Why is this important?
T3 uptake measures- the #of free binding sites in the Thyroxine Binding Globulin.
This is important because it indirectly gives the amount of thyroxine in the serum
sample.
53. What is the free thyroxine index? T7? T12? How are they calculated? What is the
importance of this calculation.
Free thyroxine index- estimates how much free T4 is in the blood.
T7 and T12- are pseudonyms for FTI (T7 =T3+T4, and T12 =T3 x T4)
FTI- an index of thyroid status, it provides T4 and T3U which are useful in
diagnosing thyroid problems.
Free thyroxine index =(T4) x (% T3 uptake as a decimal)
Page 40

54. Discuss the purpose of each of the following steps in hormone determinations:
hydrolysis- remove and solubilize attachments (chemically or enzymatically)
purification- organic solvents purify the hormones
extraction- centrifugation, washing, and ion exchange
estimation- react, detect, and quantitate by various methods
55. What is diurnal variation? How does this impact cortisol levels and the collection
of cortisol specimens?
Diurnal variation- levels of analytes rise and fall, they peak early in the day when
most of us are asleep.
Cortisol levels and ACTH (anterior pitutitary hormone, Cortocotropin)
They rise between 0600 and 0800 hours, then decrease all day long. At 2000 hrs
(8 pm), the level is 2/3 of what it is at 0800 hrs, so the best analysis is made form
early morning specimens.
56. Describe Cushing's Syndrome and Addison's disease. Include their impact on
diurnal variation, cortisol levels, glucose levels, aldosterone levels, electrolyte
and water balance. How may secondary disease states be diagnosed?
Cushinns syndrome- Increased- cortisol, diurnal variation, aldosterone,
hypertension, and hyperglycemia (glucose), and decreased potassium.
Addisons disease- Decreased- cortisol, diurnal variation, aldoserone, hypotension,
and hypoglycemia (glucose). Increased- potassium.
Addisons disease- Cosyntropin is given to th epatient which causes the cortisol
release, which aids in determining if patient has Addisons disease, because you
look to see if cotrisol is released by the cortisol stimulating drug. If still no release
of cortisol, it is a primary disease like addisons.
Cushings disease- use hiht dexamethasome suppression test where there is a
suppression of urine and plasma cortisol which only occurs in cushings disease.
57. When are plasma epinephrine and norepinephrine levels useful?
Epinephrine and Norepinephrine levels are useful in diagnosing stress, increased
catacholamines, decreased blood pressure, decreased blood volume, thyroid
hormone deficiency, and congestive heart failure.
Decreased catacholimines are found in hypotension.
58. When is it valuable to quantify HCG? What trimester of pregnancy are HCG
levels used to monitor fetal health?
It is valuable to quantitate HCG to indicate how far along in pregnancy a patient is,
or if pregnant at all. Fetal health can be determined in the 1st and 3rd trimesters by
looking at BHCG levels.
59. What is the function of serotonin? When does it increase? What is 5-hydroxy
indolacetic acid? How is 5-HIAA measured?
Serotonin is released during coagulation by platelets and is involved in smooth
muscle stimulation and vasoconstriction. It is increased with carcinoid tumors that
Page 41

occur in the Ileum and appendix, 5-HIAA is a metabolite of serotonin and is
excreted in the urine. It is measured colorimetrically after reacting it with 1-nitroso-
2-napthol and nitrois acid (purple color).
60. Why are estriol levels measured? What trimester of pregnancy are these levels
useful in?
Estriol levels are measured to help determine pregnancies. Levels in non pregnant
females can be measured in the ugs, but pregnant women have levels in the mg
range. These levels are useful in the third trimester (last 4-6 weeks of pregnancy)
M. TOXICOLOGY
1. List methods in which urines are screened for drugs of abuse? Why is urine
preferred? What is the purpose of extraction?
You can use thin layer chromatography, gas chromatography, and immunoassays
to screen for drugs of abuse. Urine is used most often because the drugs are
filtered through the kidneys and show up in the urine.
2. Why are chloramphenicol levels monitored? What disease process may result
from chloramphenicol.
Chloramphenicol levels are monitored to make sure that ALA synthase is being
produced in enough quantity that heme synthase can be produced because
chloramphenicol causes a decrease in ALA synthase, heme synthatase, and DNA
synthase. This can be associated with disease processes like sideroblastic
anemia.
3. What is the principle of the renish heavy metal test? What disease process is
associated with lead poisoning?
The principal of the renisch heavy metal is to place a clean coiled copper wire in a
solution of 5-10 ml of gastric acid or urine with an equal amount of 2M HCL, then
place in a hot water bath for 10 minutes, let sit one hour, and examine the copper
wire for color change. ( blue or purple black- antimony; dull black- arsenic; shiny
black- bismuth; and silver gray- mercury) Lead poisioning is associated with
encephalopathy characterized by cerebral edema and hypoxia.
4. How is carbon monoxide poisoning usually detected?
This is usually detected by spot tests looking for carboxyhemoglobin (giving the
patient a cherry red appearance due to its color), or gas chromatography also
looking for carboxyhemoglobin.
5. When does bromide toxicity result? How is it measured? How does bromide
affect chloride determinations?
This toxicity results from organic and inorganic medication. It is measured by
immunoassay or thin layer chromotography, and it gives a false high in chloride
determinations.
Page 42

6. What is the therapeutic usage of cyclosporine? methotrexate?
Cyclosporin- this is an immunosuppressive drug that is used to suppress host vs.
graft rejection of transplant organs.
Methotrexate- this is an antineoplastic drug that is used in therapy and involves the
rate of mitosis in normal cells versus neoplastic cells.
7. What are digoxin? digitoxin? Why are their levels so critical?
Digoxin- one of a group of cardiac glycosides obtained from digitalis plants which
restores the force of cardiac contraction in congestive heart failure. (drug used to
treat cardiovascular problems)
Digitoxin- another cardiac glycoside (less common than digoxin) that is used to
treat cardiovascular problems.
Critical levels-
o Low- digoxin caused the atrium to be less electrically excitable
o Moderate- reduce the rate of depolarization in the spontaneously
depolarizing conductive fibers.
o High- diminishes the depolarization of the ventricular myocardium.
8. Define:
therapeutic range- concentration range of a drug which is beneficial to the patient
without being toxic.
peak level- one hour after the dose is given when the drug reaches peak
concentration in the body.
trough level- the lowest concentration of drug obtained in the blood, drawn
immediately prior to the next dose.
toxic value- drug levels outside of the therapeutic range.
bioavailability- tge fraction of a drug that is absorbed into the systemic circulation.
9. What is theophyline? When is it used?
A bronchodialator used to treat asthma or other chronic obstructive pulmonary
diseases
N. VITAMINS

1. What disease process is associated with decreased B12 and folic acid? What is
the relationship between B12 and folic acid?
Pernicious anemia is associated with decreases B12 and folic acid.
In relationship between B12 an folic acid, B12 is used in the metabolism and
needed for the synthesis of folate which is needed for the production of nucleic
acids (DNA)
Page 43

2. What is the function of Vitamin A? How is it related to beta-carotene? How are
both measured? Why are serum beta carotene levels measured?
Vitamin A functions in growth, dim light vision, reproduction, immunity and mucous
secretion. Beta-carotene(pro-vitamin A) is the precursor to vitamin A and is
composed of two moles of vitamin A.
They are both measured by immunoassay or HPLC.
Beta carotene is measured in serum to indirectly quantitate 4. (P) Describe
the methods for measuring Vitamin B12? What is the function of
cyanocobalamin?



Page 44

BLOOD GASES
Purpose
Represents the acid/base status of entire body
Provides information of lung function

Sample type
Whole Blood
Arterial Sample ABG
Preferred sample
Sites are radial, femoral or brachial artery
Venous & Capillary Blood
Can be used, but not preferred

Assessment performed STAT


SPECIMEN COLLECTION & HANDLING
Collected in heparinized plastic syringe
(no air bubbles & no clots!!!)

Often Collected by Respiratory Therapy

Collected anaerobically and put on ice. Ice
serves to slow cell metabolism.

Testing performed at 37
o

C, to emulate body
temperature

PREANALYTICAL CONSIDERATIONS
Air bubbles
Causes increases in pO
2
, pH
Causes decreased in pCO
2
Clots
Can not run clotted whole blood on instrumentation
Glycolysis
Cell respiration causes a decrease in pH, pO
2
pCO
2
increases
Temperature
pH is temperature dependent. For every 1 degree rise in
temperature, the pH decreases about 0.015 units



REFERENCE VALUES (ABG)


Component Arterial Blood Mixed Venous Blood
pH 7.35-7.45 7.31-7.41
pO
2
80-100 mmHg 35-40 mmHg
O
2
Saturation > 95% 70-75%
pCO
2
35-45 mmHg 41-51 mmHg
HCO
3
-
22-26 mEq/L 22-26 mEq/L
Total CO
2
23-27 mmol/L 23-27 mmol/L
Base excess -2 to +2 -2 to +2

INSTRUMENTATION
Electrochemistry
Ion Selective Electrodes
Hemoglobin Concentration
Spectrophotometry
DETERMINATION
Three components are directly measured
pH
pO
2
pCO
2

Values that can be calculated and reported include:
Total CO
2
or bicarbonate ion
Base excess
Oxygen saturation
PH MEASUREMENT
Measure of the hydrogen ion activity based on
bicarbonate-carbonic acid buffer system

pH electrode has a thin membrane of glass
separating two differing H
+
concentrations, a H
+

exchange occurs in the outer layers of the glass,
causing a potential to develop.
A calomel half-cell or reference electrode is also
immersed in the solution.
Both the pH and reference electrode are connected
through a pH meter. The meter can measure
voltage difference between the two and convert to
pH units.
PO
2
MEASUREMENT
Partial pressure of oxygen in the blood
Measured by the O
2
electrode to determine oxygen
content
pO
2
electrode or Clark electrode measures the
current that flows when a constant voltage is
applied to the system
As dissolved O
2
diffuses from the blood a change in
current occurs which offers a direct pO
2
measurement


PCO
2
MEASUREMENT

Partial pressure of carbon dioxide in the blood
pCO
2
measured in mmHg x 0.03 indicates carbonic
acid (H
2
CO
3
)

pCO
2
> 50 mmHg =HYPO ventilation

pCO
2
< 30 mmHg= HYPER ventilation
PCO
2
MEASUREMENT
The pCO
2
electrode or Severinghaus electrode
consists of a pH electrode with a CO
2
permeable
membrane covering the glass surface. Between the
two is a thin layer of dilute bicarbonate buffer.
Once the blood contacts the membrane and the
CO
2
diffuses into the buffer, the pH of the buffer is
lowered
Change in pH is proportional to the concentration of
dissolved CO
2
in the blood

Siggaard-
Anderson
nomogram
CALCULATED PARAMETERS
Siggaard-Anderson nomogram
Base Excess
Total CO2 and bicarbonate concentration
BASE EXCESS
Determination of amount of base in the blood
Determines the source of acid-base disturbance
Base deficit usually indicates metabolic acidosis

Causes of:
Excess bicarbonate
Deficit of bicarbonate
O
2
SATURATION


Calculation/Derived
Requires measured pH and pO2 values
Measured
Requires a hgb measurement usually obtained by co-
oximetry
Co-oximetry: measuring at multiple wavelengths to get light
absorption spectra

REFERENCES
Bishop, M., Fody, E., & Schoeff, l. (2010). Clinical Chemistry:
Techniques, principles, Correlations. Baltimore: Wolters
Kluwer Lippincott Williams & Wilkins.
Carreiro-Lewandowski, E. (2008). Blood Gas Analysis and
Interpretation. Denver, Colorado: Colorado Association for
Continuing Medical Laboratory Education, Inc.
J arreau, P. (2005). Clinical Laboratory Science Review (3rd
ed.). New Orleans, LA: LSU Health Science Center.
Sunheimer, R., & Graves, L. (2010). Clinical Laboratory
Chemistry. Upper Saddle River: Pearson .


16
ELECTROLYTES

Electrolytes
Substances whose molecules dissociate into ions
when they are placed in water.
Osmotically active particles
Classification of ions: by charge
CATIONS (+)
In an electrical field, move toward the cathode
Sodium (Na), Potassium (K), Calcium(Ca), Magnesium(Mg)
ANIONS (-)
In an electrical field, move toward the anode
Chloride(Cl), Bicarbonate, PO
4
, Sulfate


2
ELECTROLYTES
General dietary requirements
Most need to be consumed only in small
amounts as utilized
Excessive intake leads to increased excretion via
kidneys
Excessive loss may result in need for corrective
therapy
loss due to vomiting / diarrhea; therapy required - IV
replacement, Pedilyte, etc.


3
ELECTROLYTE FUNCTIONS
Volume and osmotic regulation
Myocardial rhythm and contractility
Cofactors in enzyme activation
Regulation of ATPase ion pumps
Acid-base balance
Blood coagulation
Neuromuscular excitability
Production of ATP from glucose

4
ELECTROLYTE PANEL
Panel consists of:
sodium (Na)
potassium (K)
chloride (Cl)
bicarbonate CO
2
(in its ion form = HCO
3
-
)

5
ANALYTES OF THE ELECTROLYTE PANEL
Sodium (Na)
the major cation of extracellular fluid
Most abundant (90 %) extracellular cation
Diet
Easily absorbed from many foods


6
FUNCTION: SODIUM
Influence on regulation of body water
Osmotic activity
Sodium determines osmotic activity
Main contributor to plasma osmolality
Neuromuscular excitability
extremes in concentration can result in neuromuscular
symptoms
Na-K ATP-ase Pump
pumps Na out and K into cells
Without this active transport pump, the cells would fill with
Na
+
and subsequent osmotic pressure would rupture the
cells


7
REGULATION OF SODIUM
Concentration depends on:
intake of water in response to thirst
excretion of water due to blood volume or osmolality
changes
Renal regulation of sodium
Kidneys can conserve or excrete Na+ depending on ECF
and blood volume
by aldosterone
and the renin-angiotensin system
this system will stimulate the adrenal cortex to secrete
aldosterone.


8
REFERENCE RANGES:
SODIUM
Serum
136-145 mEq/L or mmol/L
Urine (24 hour collection)
40-220 mEq/L

9
SODIUM
Urine testing & calculation:
Because levels are often increased, a dilution of the
urine specimen is usually required.

Once a number is obtained, it is multiplied by the
dilution factor and reported as (mEq/L or mmol/L) in
24 hr.

10
DISORDERS OF SODIUM HOMEOSTASIS
Hyponatremia: < 136 mmol/L
Causes of:
Increased Na
+
loss
Increased water retention
Water imbalance
Hypernatremia:> 150 mmol/L
Causes of:
Excess water loss
Increased intake/retention
Decreased water intake



11
HYPONATREMIA
1. Increased Na
+
loss
Aldosterone deficiency
hypoadrenalism
Diabetes mellitus
In acidosis of diabetes, Na is excreted with
ketones
Potassium depletion
K normally excreted , if none, then Na
Loss of gastric contents

12
HYPONATREMIA
2. Increased water retention
Dilution of plasma Na+
Renal failure
Nephrotic syndrome
Hepatic cirrhosis
Congestive heart failure

13
HYPONATREMIA
3. Water imbalance
Excess water intake
Chronic condition
14
SODIUM
Note:
Increased lipids or proteins may cause false
decrease in results. This would be classified as
artifactual/pseudo-hyponatremia
15
CLINICAL SYMPTOMS OF HYPONATREMIA
Depends on the serum level
Can affect
GI tract
Neurological
Nausea, vomiting, headache, seizures,coma
16
HYPERNATREMIA
1. Excess water loss
Sweating
Diarrhea
Burns
Diabetes insipidus

17
HYPERNATREMIA
2. Increased intake/retention
Excessive IV therapy

3. Decreased water intake
Elderly
Infants
Mental impairment

18
CLINICAL SYMPTOMS OF HYPERNATREMIA
Involve the CNS
Altered mental status
Lethargy
Irritability
Vomiting
Nausea

19
SPECIMEN COLLECTION: SODIUM
Serum (sl hemolysis is OK, but not gross)
Heparinized plasma
Timed and random urine
Sweat
GI fluids
Liquid feces (would be only time of excessive
loss)
20
ANALYTES OF THE ELECTROLYTE PANEL
Potassium (K
+
)
the major cation of intracellular fluid
Only 2 % of potassium is in the plasma
Potassium concentration inside cells is 20 X greater than it is
outside.
This is maintained by the Na-K pump
exchanges 3 Na for 1 K
Diet
easily consumed by food products such as bananas
21
FUNCTION: POTASSIUM
Critically important to the functions of
neuromuscular cells
Acid-base balance
Intracellular fluid volume
Controls heart muscle contraction
Promotes muscular excitability
Decreased potassium decreases
excitability (paralysis and arrhythmias)



22
REGULATION OF POTASSIUM
Kidneys
Responsible for regulation. Potassium is readily
excreted, but gets reabsorbed in the proximal tubule -
under the control of ALDOSTERONE
Diet
Cell Uptake/Exchange

23
REFERENCE RANGES:
POTASSIUM
Serum (adults)
3.5 - 5.1 mEq/L or mmol/L
Newborns
3.7 - 5.9 mEq/L
Urine (24 hour collection)
25 - 125 mEq/L
24
DISORDERS OF POTASSIUM
HOMEOSTASIS
Hypokalemia
<3.5 mmol/L
Causes of:
Non-renal loss
Renal Loss
Cellular Shift
Decreased intake
Hyperkalemia
>5.1 mmol/L
Causes of
Decreased renal excretion
Cellular shift
Increased intake
Artifactual/False elevations


25
HYPOKALEMIA
1. Non-renal loss
Excessive fluid loss ( diarrhea, vomiting,
diuretics )
Increased Aldosterone promote Na reabsorption
K is excreted in its place

26
HYPOKALEMIA
2. Renal Loss
Nephritis, renal tubular acidosis,
hyperaldosteronism, Cushings
Syndrome

3. Cellular Shift
Alkalosis, insulin overdose

4. Decreased intake

27
MECHANISM OF HYPOKALEMIA
Increased plasma pH ( decreased Hydrogen ion )

28
K
+
moves into RBCs to preserve electrical balance,
causing plasma potassium to decrease.
( Sodium also shows a slight decrease )
H
+

K
+

RBC
CLINICAL SYMPTOMS OF HYPOKALEMIA
Neuromuscular weakness
Cardiac arrhythmia
Constipation
29
HYPERKALEMIA
1. Decreased renal excretion
Renal disease
Addisons disease
Hypoaldosteronism
2. Cellular Shift
Such as acidosis, chemotherapy, leukemia,
muscle/cellular injury
Hydrogen ions compete with potassium to get into
the cells

30
HYPERKALEMIA
3. Increased intake
Insulin IVs promote rapid cellular potassium uptake

4. Artifactual
Sample hemolysis
Prolonged tourniquet use
Excessive fist clenching


31
CLINICAL SYMPTOMS OF HYPERKALEMIA
Muscle weakness
Tingling
Numbness
Mental confusion
Cardiac arrhythmias
Cardiac arrest
32
SPECIMEN COLLECTION:POTASSIUM
Non-hemolyzed serum
heparinized plasma
24 hr urine
33
ANALYTES OF THE ELECTROLYTE PANEL
Chloride (Cl
-
)
the major anion of extracellular fluid

Chloride moves passively with Na
+
or against
HCO
3
-
to maintain neutral electrical charge

Chloride usually follows Na
if one is abnormal, so is the other
34
FUNCTION: CHLORIDE

Body hydration/water balance
Osmotic pressure
Electrical neutrality

35
REGULATION OF CHLORIDE
Regulation via diet and kidneys
In the kidney, Cl is reabsorbed in the renal proximal
tubules, along with sodium.
Deficiencies of either one limits the reabsorption of the
other.


36
REFERENCE RANGES: CHLORIDE
Serum
98 -107 mEq/L or mmol/L

24 hour urine
110-250 mEq/L
varies with intake

CSF
120 - 132 mEq/L
Often CSF Cl is decreased when CSF protein is increased,
as often occurs in bacterial meningitis.

37
DETERMINATION: CHLORIDE
Specimen type
Serum
Plasma
24 hour urine
CSF
Sweat
Sweat Chloride Test
Used to identify cystic fibrosis patients
Increased salt concentration in sweat
Pilocarpine=chemical used to stimulate sweat production
Iontophoresis=mild electrical current that stimulates sweat
production
DISORDERS OF CHLORIDE HOMEOSTASIS
Hypochloremia
Decreased blood chloride
Causes of :
Conditions where output exceeds input

Hyperchloremia
Increased blood chloride
Causes of:
Conditions where input exceeds output
39
HYPOCHLOREMIA
Decreased serum Cl
loss of gastric HCl
salt loosing renal diseases
metabolic alkalosis/compensated respiratory acidosis
increased HCO3
-
& decreased Cl
-

40
HYPERCHLOREMIA
Increased serum Cl
dehydration (relative increase)
excessive intake (IV)
congestive heart failure
renal tubular disease
metabolic acidosis
decreased HCO3- & increased Cl-
41
SPECIMEN COLLECTION: CHLORIDE
Serum
Heparinized plasma
24 hr urine
Sweat
42
ANALYTES OF THE ELECTROLYTE PANEL
Carbon dioxide/bicarbonate (HCO
3
-
)
the major anion of intracellular fluid
2
nd
most abundant anion of extracellular fluid
Total plasma CO
2
= HCO
3
-
+ H
2
CO
3
-
+ CO
2

HCO
3
-
(bicarbonate ion)
accounts for 90% of total plasma CO
2

H
2
CO
3
-
(carbonic acid)

43
FUNCTION:
BICARBONATE ION
CO2 is a waste product
continuously produced as a result of cell metabolism,
the ability of the bicarbonate ion to accept a hydrogen
ion makes it an efficient and effective means of
buffering body pH
dominant buffering system of plasma

44
REGULATION OF
BICARBONATE ION
Bicarbonate is regulated by
secretion / reabsorption of the renal
tubules
Acidosis : renal excretion
Alkalosis : renal excretion


45



REGULATION OF BICARBONATE
ION
Kidney regulation requires the enzyme carbonic anhydrase -
which is present in renal tubular cells & RBCs
carbonic anhydrase

Reaction: CO2 + H2O H2CO3 H
+
+ HCO

3

46
Pulmonary Control
Renal
Control
REFERENCE RANGE:
BICARBONATE ION
Total Carbon dioxide (venous)
23-29 mEq/L or mmol/L
includes bicarb, dissolved & undissociated H2CO3 - carbonic
acid (bicarbonate)

Bicarbonate ion (HCO3)
22-26 mEq/L or mmol/L

47
SPECIMEN COLLECTION: BICARBONATE ION
heparinized plasma
arterial whole blood
fresh serum
Anaerobic collection preferred
48
ELECTROLYTE BALANCE

Anion gap an estimate of the
unmeasured anion concentrations such
as sulfate, phosphate, and various
organic acids.

49
ELECTROLYTE SUMMARY

50
cations (+)
Na 142
K 5
Ca 5
Mg 2
154 mEq/L
anions (-)
Cl 105
HCO3- 24
HPO4- 22
SO4-2 1
organic acids 6
proteins 16
154 mEq/L
ANION GAP
Anion Gap Calculations

1. Na - (Cl + CO2 or HCO3-)

Reference range: 7-16 mEq/L

Or



2. (Na + K) - (Cl + CO2 or HCO3-)

Reference range: 10-20 mEq/L


51
FUNCTIONS OF THE ANION GAP
Causes in normal patients
what causes the anion gap?

2/3 plasma proteins & 1/3 phosphate& sulfate ions, along
with organic acids

Increased AG

uncontrolled diabetes (due to lactic & keto acids)
severe renal disorders
Hypernatremia
lab error

Decreased AG -

a decrease AG is rare, more often it occurs when one
test/instrument error


52
REFERENCES
Bishop, M., Fody, E., & Schoeff, l. (2010). Clinical Chemistry:
Techniques, principles, Correlations. Baltimore: Wolters
Kluwer Lippincott Williams & Wilkins.
http://thejunction.net/2009/04/11/the-how-to-authority-for-
donating-blood-plasma/
http://www.nlm.nih.gov/medlineplus/ency/article/002350.htm
Sunheimer, R., & Graves, L. (2010). Clinical Laboratory
Chemistry. Upper Saddle River: Pearson .


53
Water Balance
Water
60% of total body mass

Main Compartments
Intracellular (ICF)
inside cells
2/3

Extracellular (ECF)
outside cells
1/3




2
Water Balance



3
Extracellular
compartment
More on the ECF
Extracellular Compartment(ECF)
Composed of two sub-
compartments
Interstitial fluid (ISF)
Surrounds cells in tissue
Intravascular fluid (IVF)
Volume of measurable
fluid
plasma

4
Body Fluid Composition
Plasma
55% of total blood
volume
Analytes measured
directly
Consists of ions,
molecules, proteins
Serum

5
Water Balance

Ions exist in all of these fluids, but the
concentration varies depending on individual
ion and compartment
The body uses active and passive(diffusion)
transport principles to keep water and ion
concentration in place


6
Water Balance
Plasma proteins
ALBUMIN
Draw water INTO the vessels
Hydrostatic pressure
Drives water OUT of the vessels
These two forces create OSMOTIC or
ONCOTIC PRESSURE
7
Water balance
Sodium has a pulling effect on water
More Na outside cells than inside, the water is pulled out of cells
into the extracellular fluid.
Na
+
determines osmotic pressure of extracellular fluid

Proteins (especially albumin) inside the capillaries strongly pulls/keeps
water inside the vascular system
Albumin provides oncotic pressure.
By keeping Na
+
& albumin in their place, the body is able to regulate
its hydration.

When there is a disturbance in osmolality,
the body responds by regulating water intake and urinary control of
water loss or retention, not by changing electrolyte balance


8
Water Balance & Osmolality
Osmolality -
Physical property of a solution based
on solute concentration
Water concentration is regulated by
thirst and urine output
Thirst and urine production are
regulated by plasma osmolality



9
Water Balance & Osmolality
Increased osmolality stimulates two
responses that regulate water
Hypothalamus stimulates the sensation of
thirst
Posterior pituitary secrets arginine
vasopressin hormone (AVP)
AVP increases H
2
O re-absorption by renal
collection ducts

In both cases, plasma water increases


10
Osmolality
Osmolality
concentration of solute / kg
reported as mOsm / kg

another term:
Osmolarity - mOsm / L - not often used

11
Osmolality
Calculated osmolality
uses glucose, BUN, & Na values

Formula:
2 (Na) + glucose18 + BUN2.8 = calculated osmolality

Osmolal gap
Difference between calculated and determined osmolality

Formula:
Determined Osm/kg-calculated Osm/ kg= osmolal gap

Should be less than 10-15 units difference
(

12
Formulas in Action
A 40-year-old woman suffers from vomiting and
diarrhea. What would be her osmolality based on
the below data?
Sodium= 145 mmol/L
Glucose= 750 mg/ dL
BUN= 25 mg/dL


13
Regulation of Blood
Volume
Renin-angiotension-aldosterone
system
Aldosterone stimulates sodium
reabsorption and potassium ion
secretion
Natriuretic peptides
Glomerular filtration rate
Volume receptors
14
Renin-Angiotensin-Aldosterone
System
Series of events
Body detects decreased
blood volume
Renin converts
angiotensinogen to
angiotension I
Angiotension I converted
to angiotension II by ACE
Angiotension II causes
Vasoconstriction
Secretion of
aldosterone
Stimulates AVP
secretion and thirst
Enhances NaCl
reabsorption
15
References
Bishop, M., Fody, E., & Schoeff, l. (2010). Clinical Chemistry:
Techniques, principles, Correlations. Baltimore: Wolters
Kluwer Lippincott Williams & Wilkins.
http://thejunction.net/2009/04/11/the-how-to-authority-
for-donating-blood-plasma/
http://www.nlm.nih.gov/medlineplus/ency/article/002350.h
tm
Sunheimer, R., & Graves, L. (2010). Clinical Laboratory
Chemistry. Upper Saddle River: Pearson .


16
Overview of Iron
Essential mineral to most living organisms
Most abundant trace element
2-2.5 of the 3-5 grams of iron in our bodies is
found in hemoglobin (RBCs and RBC
precursors)

Where does iron come from?
Dietary sources - meats,
especially organ meats,
spinach, beats,... etc.

Regulation
Dietary sources
Absorption
Must be in ferrous state (Fe
++
)
Occurs in the stomach/small intestines
Iron stores
Iron is recycled when RBCs are broken down
25% stored in liver, spleen and bone marrow as
ferritin or (Fe
3+
)
Functions of Iron
Essential element of heme and hemoglobin
Component of methemoglobin, myoglobin
and some enzymes
Cellular oxidative mechanisms
Heme Sythesis Review
The addition of
ferrous iron
(Fe
++
)forms
heme
Forms of Iron
Ferrous(Fe
2+
)
Absorbed form
Ferric (Fe
3+
)
Ferritin
Transport and storage form
Free ferric form is picked up in the plasma by protein transferrin
Delivered to cells having receptor sites
Gut mucosal cells
Liver cells
RE system cells
Once inside the cell, ferric iron attaches to protein apoferritin to form ferritin
Deficiency of apoferritin results in ferric iron deposits or hemosiderin, which is
insoluble
8
Iron Links
http://www.umm.edu/blood/aneiron.htm

http://www.ehendrick.org/healthy/000772.htm

http://www.nlm.nih.gov/medlineplus/ency/article/000584.htm

http://www.healthservices.gov.bc.ca/msp/protoguides/gps/ferritin.html
Hemoglobin
Structure, Synthesis, Degradation and Role
Refer to Hematology notes for review
Chapter 6 in McKenzie text
Porphyrins
General structure
Cyclic compounds called
tetrapyrroles
Linked by four pyrrole
rings bonded by
methene bridges
Porphyrins
Chemical intermediates in the synthesis of
hemoglobin, myoglobin and other respiratory
pigments (cytochromes)
Clinical significance
Presence indicates abnormal heme synthesis
Physical properties
Color
Coloration around 405 nm
Usually red
Fluorescence
around 620 nm
Reddish-pink color
Chelation
Arrangement of nitrogen atoms allows chelation of metal
atoms such as iron, that participate in oxidative
metabolism
Porphyrin Synthesis & Control
Synthesis
Bone marrow and liver are the main site
Some steps of synthesis occur in mitochrondria and cytoplasm of cell

Control
Enzyme: -aminolevulinic acid (ALA)
Found in liver
Increases in hepatic heme decrease the production of ALA
Decreases or depletions of heme result in ALA increased production
Rate of heme syntheis is flexible and can change rapidily in response
to external stimuli
Porphyrins: Ones to keep an Eye on
Uroporphyrin: URO
Water soluble
Heme precursor
Found in urine
Coproporphyrin: COPRO
Water soluble
Heme precursor
Found in urine and feces
Protoporphyrin: PROTO
Water insoluble
Heme precursor
Found in feces
Porphyrinogens
Reduced form of porphyrins
Functional precursor of heme
Difficult to measure due to instability and
colorlessness
Glycated hemoglobin
Hemoglobin A
1c
most stable
Indicator of long-term glucose control
Why?
Reflects sustained average plasma glucose over the RBC
life span
Correlates with risk of cardiovascular disease and
other vascular disorders
Myoglobin
Heme protein found in skeletal and cardiac muscle
Unable to release oxygen, except under low oxygen
tension
Main function is to transport oxygen from the muscle
cell membrane to the mitochondria
Serves as an extra reserve of oxygen to help
exercising muscle maintain activity longer
Used to diagnose acute myocardial infarction
Lead
Found in the environment and in paint
Considered a toxin, plays no known role in NORMAL human
physiology
Exposure primarily respiratory or gastrointestinal

Half-life in whole blood= 2-3 weeks
Half-life= the time required by the body, tissue or organ to
metabolize or inactivate half the amount of substance
taken in

Lead
Absorption
Depends on age, nutritional status and other substances that are
present
Transport
Once in the blood, 94% transferred to RBC bound to hgb
Once it reaches its half-life, lead is distributed to soft tissues, such as
kidneys, liver and brain. Final storage is in soft tissue(5%) and bone
(95%)
Excretion
Urine (76%)
Feces (16%)
Other (8%)

Specimen Requirements: Iron
Studies
Serum without anticoagulant
Plasma with heparin ( check product
insert)
Oxalate, citrate or EDTA binds Fe ions, so
they are unacceptable
Early morning sample preferred due to
diurnal variation
No hemolysis
Iron Study/Profiles

% Fe Saturation
Total Iron
TIBC
x =

100
3
Three Components
Total Iron ( serum )
TIBC
% Iron Saturation ( Fe Sat )




The Iron Saturation is a measurement of how full
transferrin is
Assessing Iron Levels and
Forms
Directly measured
Iron
Transferrin
Beta globulin formed in the liver
Measured by the amount of iron it can bind
Ferritin
Best diagnostic test for IDA
Acute phase reactant
Assessing Iron Levels and
Forms
Indirect measure
TIBC (Total iron-binding capacity)
Measures the total amount of iron that
apotransferrin can bind
Can be expressed as a percentage(percent
saturation)
Ratio of serum iron to TIBC
Increased
Late pregnancy
IDA
Following hemorrhage
Following destruction of liver cells
Decreased
Decreased synthesis of transferrin
Increased loss of urine proteins

Test Methodologies: Iron
Colorimetric Procedure
Separate Fe from transferrin with a strong
acid
Iron is reduced from ferrous(Fe
3+
)

to
ferric(Fe
2+
) state
Addition of a chromogen creates a
colored compound
Measurement of colored product by
spectrophotometry
Iron Reference Ranges

Diurnal variation

Men: 65-165 g/dL

Women: 45-160
g/dL
Decreased Levels
Decreased intake
Increased need
Increased loss
Increased Levels
Increased absorption
Hemolytic anemia
Lead poisoning
Pernicious anemia
Megaloblastic anemia
Hepatitis

Test Methodologies:TIBC
Pre-treatment and Colorimetric Method
1. Add Fe
3+
to saturate binding sites on
transferrin
2. MgCO3 is added to remove
unbound Fe
3+

3. Mixture is centrifuged and the
supernatant tested using the serum
iron methodology
Reference Ranges
Transferrin
200-360 mg/dL
Ferritin
Male: 20-250 ng/mL
Female: 10-120 ng/mL
TIBC
250-425 g/dL
% saturation
15-55
Test Methodology:
Hemoglobin
Electrophoresis
Discussed in separate unit
Test Methodology: Porphyrins
Screening tests
Urinary PBG
Urinary ALA
Urinayr porphyrins
Quantitative Assays
URO
PROTO
COPRO
Serve to classify porphyrias
Lab Methods
Watson-Schwartz for Urinary PBG(
porphobilinogen)
Screen for acute intermittent porphyria
Specimen
Qualitative: fresh morning urine
Quantitative: 24 hour collection
Reference Range
<2 mg/daily


Watson-Schwartz
Principle
PBG + Ehrlichs
reagent results in a
red-orange
chromogen
Interferences
Urobilinogen
indole

Lab Methods: HgbA1c
Electrophoresis
Enzymatic Assays
HPLC
Goal is to separate hemoglobin forms within
a column. Then, glycated versus total
hemoglobin can be measured
spectrophotometrically
Specimen requirements
EDTA whole blood
Can be non-fasting
Reference range
4.0-6.0%
Lab Methods: Myoglobin
Procedures incorporate the binding of
specific antibodies to myoglobin with a
resulting chemical or physical change
that can be measured and correlated
to myoglobin concentration
Specimen requirements
Usually plasma ( check product insert)
Specimen Requirements:
Lead
Whole blood
Why? Circulating lead found in the RBC
Venous sample preferred but capillary
sample can be used ( must confirm
positive on capillary)
Royal blue top with EDTA anticoagulant
Lead-free containers
Urine

Lab Method: Lead
Test methodologies
AAS
Anodic stripping voltammetry

Reference Ranges in blood
Children< 10 g/dL
Iron Deficiency
Lab Features
Microcytic, hypochromic anemia
Anisocytosis, poikilocytosis
Total iron and Percent saturation decreased
TIBC increased
Hemosiderosis
Excessive levels of iron in storage
Hemochromatosis
Characterized by an increased rate of
absorption and less ferritin production
Excessive iron deposits in organs
Patient develops bronze color in the tissues
Total iron, percent saturation increased
TIBC decreased
Iron Status in Disease States
Condition Serum Iron Transferrin Ferritin % Saturation
IDA Decreased Increased Decreased Decreased
Iron Overdose Increased Decreased Increased Increased
Hematochromatosis Increased Slight Decrease Increased Increased
Malnutrition Decreased Decreased Decreased Variable
Chronic anemia Decreased Normal/decrease
d
Normal/increase
d
decreased
Acute liver disease Increased Variable Increased Increased
Case Scenario #1
A 40-year-old female is scheduled to have an
elective surgery. Her physician ordered a
routine CBC pre-op. The following test results
were obtained:

Test Result Reference Range
Hgb (g/dL) 10 12-16.0
Hct (%) 29.9 42-52
MCV (fL) 75 80-100
MCHC (g/dL) 30 32-36
WBC (x 10
3
/L) 6.0 4.5-11
Plts (x 10
9
/L) 200 150-450
Case Scenario #1
On review of her blood smear, the technician
noted target cells.
What other types of morphology would we
expect to see on this patient?

The physician then ordered a serum iron,
ferritin and TIBC level.
Case Scenario #1
Below are the results on the additional tests:
What is her diagnosis?

Test Result Reference Range
Serum iron ( g/dL) 20 65-165
Ferritin ( g/dL) 5 20-200
TIBC ( g/dL) 550 260-440
Hemoglobin Disorders
Refer to Hematology notes
Chapter 10: Hemoglobinopathies
Chapter 11: Thalassemia
Porphyrin Disorders= Porphyrias
Inherited or Acquired
Enzyme deficiencies resulting in
overproduction of heme precursors in bone
marrow or liver
Porphrias
Classification
Based on
Specific enzyme deficiency
Hepatic vs erythropoietic
Cutaneous vs neurologic
Porphyrias
Clinical symptoms
Cutaneous photosensitivity
Itchy skin
Hyperpigmentation
Inflammatory reaction occurs on exposure to
ultraviolet light
Neurologic abnormalities due to increased ALA
and PBG

Porphyrin Conditions
Secondary Conditions
Porphyrinuria
Increase in coproporphyrin production
Causes
Lead intoxication
Liver damage
Infection
Accelerated erythropoiesis
Porphyrinemia
Increase in erythrocytic protoporphyrin concentration
Causes
Lead intoxication
Iron deficiency
Impaired Iron absorption
Chronic infection
Myoglobin
Elevations
Acute myocardial
Renal failure
Vigorous exercise
Electric shock
Intramuscular injections
LEAD
Clinical Features
Children
CNS symptoms: headache ,clumsiness, seizures,
behavioral changes
GI symptoms: Abdominal pain, colic, constipation

Adults
Peripheral neuropathies, motor weakness, anemia
Case Scenario #2
A mother brings her active 2-year-old son to
the pediatrician for a routine visit. The
physician orders a CBC. Below are the results:

Test Result Reference Range
Hgb (g/dL) 10.2 14-17.4
Hct (%) 30.6 36-46
Case Scenario #2
The mother reports that her son has had
some constipation and abdominal pain. The
child does eat well, and the mother gives the
child a vitamin supplement, which includes
iron
The mother did mention that they live in an
older home that is in need of repainting.
The physician orders further testing
Case Scenario #2
Results of testing

Test Result Reference Range
Serum iron 120 65-165
Ferritin 150 20-200
Whole blood lead
(g/dL)
60 < 10
Erythrocyte
protoporphyrin
(g/dL)
150 17-77
What is the diagnosis?
Lead Poisoning
How does this occur?
Lead inhibits certain
enzymes in the heme
synthesis pathway

Case Scenario #2
IDA was ruled out based on the serum iron
and ferritin levels
Functions of the Heart
Pumps blood to the organs of the body
Delivers oxygen and nutrients where they are
needed
Removes waste products from tissues

Symptoms of Heart Disease
Pathologic Conditions of the Heart
Congenital Cardiovascular Defects
Abnormality arises from abnormal formation of
heart or its major blood vessels
Present at birth
All defects develop before the 10
th
week of pregnancy
Origin unknown but appear to be based on
genetic disposition and environmental influences
Congenital Cardiovascular Defects
Symptoms
Cyanosis
Pulmonary hypertension
Embolism
Clubbed fingers
Reduced growth
Syncope
Examples
Tetralogy of Fallot
Ventricular septal defects hole in the heart
Pathologic Conditions of the Heart

Heart Failure or Congestive Heart Failure
Any structural or functional cardiac disorder that impairs
the ability of the ventricle to fill with or eject blood
Result
Excess fluid accumulates in the lungs producing edema
Reduced output of blood to systemic circulation
Retention of fluid by the kidneys
Heart Failure or Congestive Heart
Failure
Examples
Left ventricular dysfunction
Coronary artery disease
Cardiac arrhythmias

See it:
http://www.youtube.com/watch?v=3cW8__wFXDA


Pathologic Conditions of the Heart
Acute Coronary Syndromes
Term used to describe a series of events
Angina
Reversible tissue injury
Unstable angina
Myocardial infarction
Extensive tissue necrosis
Acute Coronary Syndromes
Clinical Symptoms
Chest pain
Referred pain
Nausea
Vomiting
Dyspnea
Diaphoresis
Light headedness
Acute Coronary Syndromes
Causes
Atherosclerosis
Inflammatory disorder
Plaques deposit in
artery walls
Leads to ischemia
Stages of Atherosclerosis
1. Initial vascular injury caused by:
1. Hypertension, hyperlipidemia, hyperhomocysteinemia
2. Increased permeability to lipids especially LDL/VLDL
1. Results in inflammation
3. Monocytes & Leukocytes arrive to help!
4. Macrophages scavenge LDL/cholesterol-rich lipoproteins- become foam cells
5. Foam cells promote lesion progression
6. T and B lymphocytes are recruited by the plaque
7. Interactions between T/B lymphs and foam cells recruits smooth muscle cells into
the lumen
8. Smooth muscle cells secrete collagen, elastin, and proteoglycans to fix the plaque
to the vessel wall

See the process
Presentation of Coronary Heart Disease
Hypertension
Persistent systolic BP of at least 140 mm HG
and/or diastolic BP of at least 90 mm Hg
Prevalence increases with age
Contributing factors
Obesity
Physical inactivity
Unhealthy nutrition
Hypertensive Heart Disease
Term used to describe heart disease caused by
direct or indirect effects of increased BP
Peripheral resistance determining factor in BP
Increases workload of left ventricle resulting in
hypertrophy and dilation of mitral valve. This valve
is affected and blood is regurgitated to the left
atrium
Infective Heart Disease
Heart disease caused by infectious agents
Examples
Rheumatic Heart Disease
Complication of rheumatic fever due to autoimmune response.
Causative organism is Group A streptococcus
Usually affects young adults and children
Infective Endocarditits
Infection of endocardial surface of the heart
Causative organism Group D streptococcus, but others also
Pericarditis
Inflammation of the pericardium
Causative agents include bacteria, fungi, viral, autoimmune, others
Diagnosing Heart Disease
Myocardial Infarction
Diagnosis based on clinical symptoms, EKG
changes and the rise/fall of biochemical markers
Samples collected at onset, 6-9 hours and 12-24
hours if previous samples were negative
Preferred biomarkers are Troponin I and T.
Specific and sensitive for myocardial necrosis
Current guidelines suggest the use of 2 markers for
diagnosis

Current Cardiac Panel
Myoglobin
Released from damaged cardiac/skeletal muscle
Cardiac troponins
See upcoming slide
CK
CK-MB
BNP
Discussed later

Time Course Of Enzyme Activity in MIs
Historically CK, CK-MB, AST, LD/LDH
isoenzymes used
Enzyme Onset of
Elevation
Peak
activity
(Hr)
Duration
of
Elevation
CK 4-8 12-24 3-4
CK-MB 4-6 12-24 2-3
AST 8-12 24 5
LD 12-24 72 10
LDH
isoenzymes
12-24 5
Time Course Of Enzyme Activity in MIs
Troponin
Rises 4-10 hours after
onset
Peak at 12-48 hours
Elevated for 4-10 days
Myoglobin
Released 1-4 hours after
onset
CK-MB
Rises within 4-6 hours after
onset
Peaks at 12-24 hours
Normal at 2-3 days

Troponin
Consists of three proteins that bind to thin filament(actin) of
cardiac and skeletal muscle
Troponin T (TnT)
Troponin I (TnI)
Troponin C (TnC)
Function to bind Ca
+
and regulate muscle contraction
Absent in the serum of healthy people
Whats So Special About Troponin?
Specific for cardiac tissue
High diagnostic specificity and sensitivity
Early detection following MI
Remain elevated for several days
Undetected in healthy people
Few interfering substances in detection
Markers of Inflammation
High Sensitivity C-Reactive Protein (hsCRP)
Acute phase protein
Produced in the liver in response to injury,
infection and inflammation
Increases in CRP correlate with the risk of
coronary artery disease

Markers of Congestive Heart Failure
Natriuretic peptide
Hormones that include atrial natriuretic peptide
(ANP), B-type natriuretic peptide(BNP), C-type and D-
type
Assist in regulation of cardiovascular homeostasis

BNP
Released on ventricular stretch or stress to myocytes in the
absence of necrosis
Increased BNP indicates expanded fluid volume such as that
seen in renal failure and CHF
Vascular Inflammation
Plaque Destabilization
Plaque Rupture
Acute Phase Reactant (CRP)
Ischemia
Necrosis (Troponin)
Myocardial Dysfunction (BNP, NT-proBNP)
References
Bishop, M., Fody, E., & Schoeff, l. (2010). Clinical Chemistry:
Techniques, principles, Correlations. Baltimore: Wolters
Kluwer Lippincott Williams & Wilkins.
http://medicinembbs.blogspot.com/2010/12/congestive-
heart-failure.html
http://www.resverlogix.com/product_development/cardiovas
cular_disease/atherosclerosis.html
http://www.youtube.com/watch?v=upb37rbS1dE
http://www.youtube.com/watch?v=3cW8__wFXDA
http://smabiology.blogspot.com/2009_04_01_archive.html
Sunheimer, R., & Graves, L. (2010). Clinical Laboratory
Chemistry. Upper Saddle River: Pearson .

Introduction
2
Organisms rely on the oxidation of complex
organic compounds to obtain energy

Three general types of compounds provide
chemical energy to our cells
Lipids=Fats
Amino acids = Proteins
Carbohydrates= Sugars, starches

Carbohydrates
3
Major food source & energy supply of body

Primary source of energy for brain, erythrocytes, retinal cells

Depending on individual diet, 50-90% of the body's carbohydrate
intake is in the form of
Grains - cereals, bread
Starchy vegetables - potatoes
Legumes - beans, peanuts
other sources = sugar, molasses, lactose from mi lk, fructose from frui t

Stored primarily as liver and muscle glycogen



Description and Classification of Carbohydrates
Contain C, H and O molecules
Contain a C=O (ketone) and an OH(aldehyde)
functional group

Classification
Based on certain properties
The size of the base carbon chain
Location of the CO functional group
Number of sugar units
Stereochemistry of compound

Chemical Properties
5
Some ( not all ) carbs are reducing substances (donate electrons)
Chemical reduction of other substances
These sugars must contain an aldehyde or ketone group
Reducing sugars

Glucose
Maltose
Lactose
Fructose
Galactose

Sucrose is not a reducing substance


Carbohydrate Metabolism
Glucose is primary energy source
Nervous tissue can not concentrate or store
carbohydrates, so a steady supply of glucose is
needed
Once the level of glucose falls below a certain range,
normal function is impaired
Carbohydrate Breakdown
Dietary
Carbohydrates
Mouth
Salivary amylase
Dextrins/
Maltose
Stomach/Intestines
Pancreatic amylase
Monosaccharide
Absorption into intestinal mucosa
Delivered to liver


Carbohydrate Breakdown
Ultimate Goal
Convert glucose to CO
2
and water with ATP as a by-product

Possible channels
Converted to liver glycogen and stored
Metabolized to CO2 and H2O
Converted to keto-acids, amino acids, and proteins
Converted to fats and stored in adipose tissue
Biochemical Pathways in Carbohydrate Breakdown
Embden-Meyerhoff pathway
Converts glucose to pyruvate/lactate
Primary energy source for humans
Hexose monophosphate shunt
Oxidizes glucose to ribose and CO2
Produces NADPH as an energy source
Glycogenesis
Converts glucose to glycogen

Carbohydrate Metabolism
10
Glycolysis the conversion of glucose and other hexoses into
lactate or pyruvate
Breakdown of glucose for energy production

Glycogenesis the conversion of glucose to glycogen
usually in liver & muscle
Excess glucose is converted and stored as glycogen
High concentrations of glycogen in liver and skeletal muscle
Glycogen is a quickly accessible storage form of glucose

Carbohydrate Metabolism
11
Glycogenolysis the breakdown of glycogen to form glucose
Glycogenolysis occurs when plasma glucose is decreased
Occurs quickly if additional glucose is needed
Controlled by hormones & enzymes

Gluconeogenesis the formation of glucose from non-carbohydrate
sources, such as ami no aci ds, glycerol & fatty aci ds into glucose
Occurs mainly in the liver


Glucose
Glycogenesis
Glycogenolysis
Glycolysis
Gluconeogenesis
Carbohydrate Metabolism
13
Also related:
Lipogenesis the conversion of carbohydrates to fatty acids
Fat is another energy storage form, but not as quickly accessible as
glycogen
Lipolysis the decomposition of fat

The sum or net of all of these pr ocesses deter mi nes
the level of blood glucose.

Regulation of Plasma Glucose
14
Organs / systems involved in glucose regulation

Liver : Glucose Glycogen Glucose

Muscle
Skeletal & heart

Pancreas
Synthesizes hormones Insulin and Glucagon, somatostatin

Other Endocrine glands
Anterior pituitary gland ( growth hormone)
Adrenal gland (epinephrine and cortisol)
Thyroid gland (thyroxine)



Regulation of Plasma Glucose
15
If plasma glucose is decreased :
Glycogenolysis
The liver releases glucose into the plasma (quick response)
Gluconeogenesis and lipolysis

If plasma glucose is increased :

Glycogenesis
Liver stores glucose as glycogen
Lipogenesis
Formation of lipids


Hormones that Regulate Glucose

16
Insulin
Most important & only one to decrease glucose
level
Synthesized in the Beta cells of the Islets of Langerhans
(in the pancreas)
Released when plasma glucose is increased

Action / Effects of insulin

Facilitates glucose entry into cells
cell membranes need insulin to be present for glucose to enter

Promotes liver glycogenesis
glucose to glycogen

Promotes glycolysis
speeds up utilization of glucose in cells

Promotes synthesis of lipids from glucose
Such as the formation of Tri glyceri des

Promotes amino acid synthesis from glucose intermediates

Decreases / inhibits glycogenolysis and gluconeogenesis

Insulin Control
18
Insulin secretion controlled by:
Blood glucose level
Certain Amino Acids ie. leucine, & arginine



Counterregulatory Hormones
19
Glucagon
2
nd
most important glucose regulatory hormone

Referred to as a hyperglycemic agent

Synthesized in alpha cells of the islets of Langerhans




Action/Effect of Glucagon
20
Stimuli decreased plasma glucose
Action
Increases glycogenolysis & gluconeogenesis
Promotes breakdown of fatty acids
Promotes breakdown of proteins to form amino acids
Increases plasma glucose concentration


Other Regulatory Hormones
21
Epinephrine
One of two glucose regulating hormones from the adrenal gland
Origin adrenal medulla
Action/effect
Inhibits insulin secretion & release
Promotes lipolysis
Stimulates glycogenolysis
Immediate release of glucose

Stimuli
Neurogenic - based on physical / emotional stress.
Adrenal tumors
Other Regulatory Hormones
22
Glucocorticoids - such as cor ti sol
Origin adrenal cortex

Effect antagonistic to insulin
increases blood glucose
promotes gluconeogenesis from breakdown of proteins
inhibits the entry of glucose into muscle cells

Stimuli anterior pituitarys ACTH
Other Regulatory Hormones
23
Growth Hormone (GH) and Adrenocorticotropic
Hormone (ACTH)
Origin anterior pituitary gland
Effect antagonistic to insulin
Increases plasma glucose levels
inhibits insulin secretion
inhibits entry of glucose into muscle cells
inhibits glycolysis
inhibits formation of triglycerides from glucose
Stimuli
decreased glucose stimulates its release
increased glucose inhibits its release

Other Regulatory Hormones
24
Thyroid hormones (such as thyroxi ne)
Origin thyroid gland
Effect
increases absorption of glucose from intestines
Promotes comversion of liver glycogen to glucose
Stimuli pituitary glands TSH




Other Regulatory Hormones
Somatostatin
Origin-Delta cells of the islets of Langerhans in the pancreas
Effect - increase plasma glucose
Actions
antagonistic to insulin,
inhibits endocrine hormones including glucagon & growth
hormone

References
Bishop, M., Fody, E., & Schoeff, l. (2010). Clinical Chemistry:
Techniques, principles, Correlations. Baltimore: Wolters
Kluwer Lippincott Williams & Wilkins.
Sunheimer, R., & Graves, L. (2010). Clinical Laboratory
Chemistry. Upper Saddle River: Pearson .


26
Laboratory testing
Considerations
Reference values depend on:
Type of specimen
venous/capillary
Serum, plasma, whole blood
How was it collected?
fasting, random, after a meal
Reference value (serum/plasma)
74-106 mg/dL


2
Laboratory testing
Glucose preservation
Perform testing < 1 hour after collection
Separate plasma from cells < 1 hour
Cells continue to utilize glucose at a rate of 10 mg/dL per
hour.
Refrigeration slows the process.
Collect blood in sodium fluoride tube
Grey top tube
Fluoride inhibits glycolysis


3
Specimen Collection
Whole blood
Point of care
Results are @ 11% lower than
plasma/serum
Serum
Plasma



4
Other Specimen Types
CSF specimens
Analyzed ASAP
Glucose level is 60-70% of pts current blood level.
CSF glucose in Fasting (non-diabetic) @ 40-70 mg/dL
Decreased CSF glucose values suggest bacterial meningitis
because bacteria are consuming glucose as an energy source
Normal or Increased CSF glucose suggests viral meningitis.


24 hour urine
A small amount of glucose is lost in the urine daily. Usually
< 500mg/24 hr.
Random urine for diagnosis no longer performed, but
some patients use it for self monitoring.


5
Methods for Glucose
Determination
Glucose Oxidase Methodology
7



Glucose + O
2
+ H
2
O
Glucose Oxidase Gluconic acid + H
2
O
2

H
2
O
2
+ Chromogen
Oxidized chromogen
+ H
2
O
Peroxidase
Trindler reaction
Glucose oxidase an enzyme that will catalyze the
reaction of glucose to gluconic acid, with the formation
of hydrogen peroxide as a by-product
Glucose oxidase
Good methodology, but:

Procedure is good for blood and CSF specimens, but urine
has too many interfering substances.

Subject to interference from ascorbic acid, bilirubin and uric
acid which are also oxidized by peroxidase.

Alternative way to determine concentration:
(polarographically)
Measuring the amount of oxygen used up by an
electrode


8
Hexokinase
An enzyme that catalyzes the
phosphorylation of glucose
Method can be very accurate and precise since
the coupling reaction is specific
Time consuming for routine use
Reference methodology since it lacks
interferences associated with glucose oxidase
method
Procedure can utilize blood, urine and CSF


9
Hexokinase Methodology

10



Glucose + ATP
Hexokinase Glucose 6 Phosphate +
ADP


Glucose 6 - Phosphate
+ NADP
G6PD
NADPH + H +
6-Phosphogluconate
NADP - Nicotinamide adenine dinucleotide phosphate (oxidized
form) is reduced

NADPH - reduced form absorbs light (340nm) proportional to the
amount of glucose present in first reaction


Laboratory Diagnosis

Laboratory Tests
Fasting blood sugar (FBS)
Most frequently ordered screening test
for glucose metabolism
Reference value: 74-106 mg/dL
Fasting values > 126 mg/dL usually indicate a
problem
FBS should be repeated on another day to
confirm diagnosis

Borderline diabetes may have a normal FBS &
may need a challenge test to demonstrate
abnormality


12








Laboratory Tests








2 Hour Postprandial
Patient has FBS drawn
Ingests a 75 gram high carbo breakfast
or sometimes drinks glucola
Has repeated glucose test at 2 hours
Glucose level should have returned to
fasting levels.
If glucose > 200 mg/dL on the
postprandial test, a fasting or random
glucose level, should be performed on a
subsequent day to diagnose with diabetes
13
Laboratory Tests
Oral glucose tolerance test (GTT)

No longer recommended by the new ADA
guidelines
Used to screen for gestational diabetes

Problems included calculation dosage, patient
must drink it, keep it down, stay relatively
inactive during test period, and be
successfully drawn on time.


14
Oral glucose tolerance test (GTT)
Patient directions - important.
Eat an adequate carbohydrate diet at least three (3) days
prior to test
Evening before the test, no eating after supper meal
Test is begun in early a.m.
Obtain fasting specimen
Test dose: ** test dose has been reduced to 75 gm for
adults and 1.75 gm / kg for children. Test dose must be
consumed within 5 minutes.
Patient is to remain resting, no smoking or eating during
test period

Blood and urine specimens are collected at hourly intervals
- Testing of the urine glucose & ketones, no longer routine.
15
Oral glucose tolerance test (GTT)

Response to Oral Glucose Tolerance Test Response to Oral Glucose Tolerance Test
Normal
Abnormal


Laboratory Tests: Ketones


Produced by the liver
Metabolism by-products of fatty acids
Three bodies
Acetone (2%)
Acetoacetic acid (20%)
3- hydroxybutyric acid (78%)

Increase in cases of carbohydrate deprivation
or decreased carbohydrate use (diabetes
mellitus, starvation/fasting, prolonged
vomiting etc.)
Laboratory Tests:
Microalbumin
Microalbumin
Persistent albuminuria in the range of 30-299
mg/ 24 h or an albumin-creatinine ratio of 30-
300 g/mg
Indication of renal nephropathy
Assists in the diagnosis of early proteinuria
Normal urine dipsticks are insensitive to low
concentrations of urine albumin

Glycosylated Hemoglobin/
Hemoglobin A1c

Long term glycemic control indicator,
reflects average blood glucose level over
the previous 2-3 months

Glucose molecule attaches
nonenzymatically to the hemoglobin
molecule

Advantages:
Time average glucose not subject to
temporary variability due to diet and
exercise
Does not require fasting

Influenced by:
Conditions that affect the life span of the
RBC, such as sickle cell disease and
hemolytic diseases
Hemoglobin A
1C
is the most commonly
measured glycosylated hemoglobin

19
Glycosylated Hemoglobin/
Hemoglobin A1c
Specimen : EDTA whole blood
doesnt need to be fasting

Measured by electrophoresis, enzymatic assays,
HPLC

Hemoglobin A
1C
reference range
4.0 - 6.0 %

For diagnosis of diabetes based on Hemoglobin
A
1C
results, the patient must has a result of > 6.5%
, confirmed by repeat measurement.

Other related tests:
Lactose Tolerance


Lactose - disaccharide
Lactose malabsorption or lack of enzyme
needed to breakdown lactose
Often results in diarrhea, cramping, and gas

Lab evaluation
Perform OGTT using lactose, not glucose
Normal
GTT curve similar to OGTT (glucose level will
increase 25 mg/dL above the fasting level).
Lactase deficiency
Flat curve - no/very little increase in glucose level.


21
Urine Glucose
Copper Reduction- Clinitest

Not specific
Detects all reducing sugars
Used to detect galactosemia in babies
and children < 3 yrs old.

References
Bishop, M., Fody, E., & Schoeff, l. (2010). Clinical Chemistry:
Techniques, principles, Correlations. Baltimore: Wolters
Kluwer Lippincott Williams & Wilkins.
Sunheimer, R., & Graves, L. (2010). Clinical Laboratory
Chemistry. Upper Saddle River: Pearson .


23
Hyperglycemia
Increase in plasma glucose
levels due to hormone
imbalance

Healthy patients
Insulin is secreted by the
cells of the pancreatic
islets of Langerhans
Reference Range
Increased plasma glucose:
> 110 mg / dl
Glucose reference range:
74 - 106 mg / dl



2
3
Effects of Hyperglycemia
Immediate Effects
Increased extracellular osmotic pressure
The increased glucose in plasma pulls water out
of cells
Results in dehydration

Acidosis - metabolic acidosis.
May result
If the patients cells are not able to take in
glucose, they may begin to convert fats to fatty
acids, which then become keto acids.



4
Effects of Hyperglycemia: Long term
Physiological
Heart attacks/strokes, Diabetic
retinopathy(Blindness), kidney failure, neurologic
defects, susceptibility to infections

Chemical
Glycosylated hemoglobin
the formation of glycosylated hemoglobin is the result of
prolonged elevation of plasma glucose.


Diabetes
Characterized by hyperglycemia
Disorders differ in etiology, symptoms and
consequences

Labs role
Assist in diagnosis of the disease
Identification of the disorder
Assessment of progression of tissue damage
5
Physiologic abnormalities of diabetes

Hyperglycemia
increase blood glucose.
Doesnt matter how the glucose is derived - diet, fat
metabolism, protein destruction/wasting
Ketosis
from fat metabolism, ketonemia, ketonuria
Hyperlipidemia -increase blood lipids from faulty glucose
metabolism.
Decrease blood pH - metabolic acidosis
Urine abnormalities
Glycosuria glucose present
Polyuria - increase in urine volume
Loss of electrolytes - washing out with the urine

6
7
Diabetes

World Health Organization (WHO) and American
Diabetes Association (ADA) recommends four
categories of diabetes:
Type 1 diabetes
Most severe and potentially lethal
Type 2 diabetes
Other (secondary diabetes)
Gestational diabetes mellitus (GDM)
Type 1 Diabetes


Insulin dependent diabetes mellitus ( IDDM )
5-10 % of diabetes cases

Demographics
Non-Hispanic Whites/ Non-Hispanic Blacks
Children & adolescents

Pathology
Disease triggered by viral illness or environmental factors that
destroys beta cells in pancreas.
Absolute Insulin deficiency
Defect in secretion, production or action or all
Autoimmune destruction of islet beta cells in pancreas
Auto-antibodies are present

8
Type 1 Diabetes
Clinical Symptoms

CLASSIC TRIAD
Polyphagia (increased
food uptake)
Polydipsia (thirst)
Polyuria ( increased
urine production)

Other symptoms
Mental confusion
Rapid weight loss
Hyperventilation
Diabetic ketoacidosis



9
Laboratory Findings


Hyperglycemia- plasma levels > 110 mg/dL
Glucosuria- plasma glucose > 180 mg / dl

Decreased insulin
Increased glucagon
Stimulation causes
Gluconeogenesis
Lipolysis (breakdown of fat produces ketones)

Ketoacidosis
Decreased blood pH ( acidosis )
Sodium Potassium CO
2

10
Type II Diabetes

Non Insulin Dependent Diabetes Mellitus( NIDDM )

Most common form of diabetes
Demographics
Adult onset
Patients usually > 20 years old
American Indians and non-Hispanic blacks





11
Type II Diabetes: Pathology
Develops gradually
Disorder in insulin resistance and relative
deficiency of insulin
Plasma glucose is unable to enter cells

Contributory factors
Obesity
Lack of exercise
Diet
Genetics
Drugs, such as diuretics, psychoactive drugs
Increases in hormones that inhibit/antagonize
insulin (GH & cortisol)

12
Laboratory Findings
Hyperglycemia
Glucosuria
Insulin is present
Glucagon is not elevated

No lipolysis and no ketoacidosis
Excess glucose is converted to triglycerides ( plasma
triglycerides )


Normal / Increased Na / K
Increased BUN & Creatinine ( Decreased renal function )
Hyperosmolar plasma from hyperglycemia

13
Other (SecondaryDiabetes)
Genetic defects of beta cell function
Genetic defects in insulin action
Genetic syndromes
Pancreatic disease
Endocrinopathies
Drug or chemical induced

14

Glucose intolerance associated with pregnancys
hormonal and metabolic changes

Mothers usually return to normal after pregnancy, but with
increased risk for diabetes later on in life

Infants are at increased risk for respiratory complications
and hypoglycemia after birth
15
Criteria for Diagnosis of Diabetes
1. Symptoms of diabetes plus random plasma glucose concentration > 200
mg/dL. Random is defined as any time of day without regard to time
OR
2. Fasting plasma glucose > 126 mg/dL. Fasting is defined as no caloric intake
for at least 8 hours.
OR
3. 2-Hour postprandial glucose > 200 mg/dL during an oral glucose tolerance
test

OR
4. A HgbA
1
C > 6.5%, confirmed on repeat measurement

Side notes
Glucose tolerance testing ( GTT ) is considered to be of limited additional
use in the diagnosis of diabetes and not recommended, do 2 hour pp test
as stated above.
Urine glucose testing is also not recommended in diabetes diagnosis

16
17
Hypoglycemia
Plasma glucose level falls below 60 mg/dL

Glucagon is released when plasma glucose is < 70
mg / dL to inhibit insulin
Epinephrine, cortisol, and growth hormone released
from adrenal gland to increase glucose metabolism
and inhibit insulin

Treatment
Varies with cause. Generally, hypoglycemia is
treated with small, frequent meals, (5-6 / day) low
in carbohydrates, high in protein


Hypoglycemia
Symptoms
Increased hunger
Sweating
Nausea
Vomiting
Dizziness
Shaking
Blurring of speech
and sight
Mental confusion
Lab Findings
Decreased plasma
glucose
18
Whipples Triad
Symptoms of hypoglycemia
Low plasma glucose at time of
symptoms
Alleviation of symptoms with
glucose ingestion

19
20
Hypoglycemia
Causes of:
Reactive
Insulin overdose in diabetics
Ethanol ingestion
Fasting
Insulin-producing tumors
Hepatic dysfunction
Sepsis

21
Galactosemia
Resulting from :
Galactose 1, phosphate uridyl transferase deficiency
enzyme that converts galactose to glucose, patients cannot
change either galactose or lactose into glucose.
results in galactosemia (galactose in blood)

Effects:
Can lead to mental retardation, cataracts, death

check children < 3 yrs for reducing substances



References
Bishop, M., Fody, E., & Schoeff, l. (2010). Clinical Chemistry:
Techniques, principles, Correlations. Baltimore: Wolters Kluwer
Lippincott Williams & Wilkins.
Centers for Disease Control. (2012). Diabetes Public Health
Resource. Retrieved from
http://www.cdc.gov/diabetes/pubs/factsheet11.htm
Sunheimer, R., & Graves, L. (2010). Clinical Laboratory
Chemistry. Upper Saddle River: Pearson .
http://crossfitovercome.com/2011/12/29/diabetes-primer/

22
Terms
Acid
Any substance that can yield a hydrogen ion
(H
+
) or hydronium ion when dissolved in water
Release of proton or H
+


Base
Substance that can yield hydroxyl ions (OH
-
)
Accept protons or H
+

Terms
pK/ pKa
Negative log of the ionization constant of an acid
Strong acids would have a pK <3
Strong base would have a pK >9

pH
Negative log of the hydrogen ion concentration
pH= pK + log([base]/[acid])
Represents the hydrogen concentration
Terms
Buffer
Combination of a weak acid and /or a
weak base and its salt
What does it do?
Resists changes in pH

Effectiveness depends on
pK of buffering system
pH of environment in which it is placed
Terms
Acidosis
pH less than 7.35

Alkalosis
pH greater than 7.45

Note: Normal pH is 7.35-7.45
Acid-Base Balance
Function
Maintains pH homeostasis
Maintenance of H
+
concentration

Potential Problems of Acid-Base balance
Increased H
+
concentration yields decreased pH
Decreased H
+
concentration yields increased
pH
Regulation of pH
Direct relation of the production and retention of acids and
bases
Systems
Respiratory Center and Lungs
Kidneys
Buffers
Found in all body fluids
Weak acids good buffers since they can tilt a reaction
in the other direction
Strong acids are poor buffers because they make the
system more acid
8
Blood Buffer Systems
Why do we need them?
If the acids produced in the body from the
catabolism of food and other cellular
processes are not removed or buffered,
the bodys pH would drop
Significant drops in pH interferes with cell
enzyme systems.
Blood Buffer Systems
Four Major Buffer Systems
Protein Buffer systems
Amino acids
Hemoglobin Buffer system
Phosphate Buffer system
Bicarbonate-carbonic acid Buffer system
Blood Buffer Systems
Protein Buffer System
Originates from amino acids
ALBUMIN- primary protein due to high
concentration in plasma
Buffer both hydrogen ions and carbon
dioxide

Blood Buffering Systems
Hemoglobin Buffer System
Roles
Binds CO
2

Binds and transports hydrogen and
oxygen
Participates in the chloride shift
Maintains blood pH as hemoglobin
changes from oxyhemoglobin to
deoxyhemoglobin
Oxygen Dissociation Curve
Curve B: Normal
curve

Curve A: Increased
affinity for hgb, so
oxygen keep close

Curve C: Decreased
affinity for hgb, so
oxygen released to
tissues
Bohr Effect
It all about
oxygen
affinity!
Blood Buffer Systems
Phosphate Buffer System
Has a major role in the elimination of H
+

via the kidney
Assists in the exchange of sodium for
hydrogen
It participates in the following reaction
HPO
-2
4
+ H
+
H
2
PO

4

Essential within the erythrocytes

Blood Buffer Systems
Bicarbonate/carbonic acid buffer
system
Function almost instantaneously
Cells that are utilizing O
2
, produce CO
2
, which
builds up. Thus, more CO
2
is found in the tissue
cells than in nearby blood cells. This results in a
pressure (pCO
2
).
Diffusion occurs, the CO
2
leaves the tissue
through the interstitial fluid into the capillary
blood
Bicarbonate/Carbonic Acid Buffer
Carbonic
acid
Bicarbonate
Conjugate
base
Excreted in
urine
Excreted
by lungs
Bicarbonate/carbonic acid buffer system

How is CO
2
transported?
5-8% transported in dissolved form
A small amount of the CO
2
combines directly
with the hemoglobin to form
carbaminohemoglobin
92-95% of CO
2
will enter the RBC, and under
the following reaction
CO
2
+ H
2
0 H
+
+ HCO
3
-
Once bicarbonate formed, exchanged for
chloride

Henderson-Hasselbalch Equation
Relationship between pH and the
bicarbonate-carbonic acid buffer
system in plasma

Allows us to calculate pH
Henderson-Hasselbalch Equation
General Equation

pH = pK + log A
-
HA


Bicarbonate/Carbonic Acid system

o pH= pK + log HCO
3

H
2
CO
3
( PCO
2
x 0.0301)









Henderson-Hasselbalch Equation
1. pH=pK+log H
HA
2. The pCO
2
and the HCO
3
are read or derived from the blood gas analyzer
pCO
2
=40 mmHg
HCO
3
-
=24 mEq/L

3. Convert the pCO
2
to make the units the same
pCO
2
=40 mmHg * 0.03=1.2 mEq/L

3. Lets determine the pH:
4. Plug in pK of 6.1

5. Put the data in the formula
pH =pK +log 24 mEq/L
1.2 mEq/L
pH =pK +log 20
pH=pK+1.30
pH=6.1+1.30
pH=7.40


The Ratio.
Normal is : 20 = Bicarbonate = Kidney = metabolic
1 carbonic acid Lungs respiratory


The ratio of HCO
3
-
(salt/bicarbonate) to H
2
CO
3

(acid/carbonic acid) is normally 20:1

Allows blood pH of 7.40
The pH falls (acidosis) as bicarbonate
decreases in relation to carbonic acid
The pH rises (alkalosis) as bicarbonate
increases in relation to carbonic acid



Physiologic Buffer Systems
Lungs/respiratory
Quickest way to respond, takes minutes
to hours to correct pH by adjusting
carbonic acid
Eliminate volatile respiratory acids such
as CO
2
Doesnt affect fixed acids like lactic acid
Body pH can be adjusted by changing
rate and depth of breathing blowing off
Provide O
2
to cells and remove CO
2



Physiologic Buffer Systems
Kidney/Metabolic
Can eliminate large amounts of acid
Can excrete base as well
Can take several hours to days to correct pH
Most effective regulator of pH


If kidney fails, pH balance fails


25
References
Bishop, M., Fody, E., & Schoeff, l. (2010). Clinical
Chemistry: Techniques, principles, Correlations. Baltimore:
Wolters Kluwer Lippincott Williams & Wilkins.
Carreiro-Lewandowski, E. (2008). Blood Gas Analysis and
Interpretation. Denver, Colorado: Colorado Association for
Continuing Medical Laboratory Education, Inc.
Sunheimer, R., & Graves, L. (2010). Clinical Laboratory
Chemistry. Upper Saddle River: Pearson .


26

Potrebbero piacerti anche